Крок 3 - Медицина 2012 весна (буклет)

1 / 200
Хворий 68-ми років скаржиться на періодичну короткочасну непритомність при різких поворотах голови. Об’єктивно: 6лідість шкіри, АТ-80/40 мм рт.ст. Тони серця ослаблені, брадикардія. Який найбільш вірогідний діагноз? A 68-year-old patient complains of periodic short-term fainting when suddenly turning the head. Objectively: 6 skin coldness, BP-80/40 mm Hg. Tons hearts are weakened, bradycardia. What is the most likely diagnosis?

Ортостатична непритомність Orthostatic syncope

Вазовагальна непритомність Vasovagal syncope

Колапс Collapse

Синоаурикулярна блокада Sino-auricular blockade

Синокаротидна непритомність Sinocarotid syncope

2 / 200
У хворої через 3 години після вечері, серед повного здоров’я, виник гострий біль у правому підребер’ї, який віддає в праву лопатку, з’явились лихоманка, блювання жовчю. Пальпаторно в правому підребер’ї болючість, напруження м’язів передньої черевної стінки. Позитивний симптом Грекова-Ортнера. Який найбільш імовірний діагноз? The patient, 3 hours after dinner, in full health, developed a sharp pain in the right hypochondrium, which radiates to the right shoulder blade, fever appeared , vomiting of bile. Pain on palpation in the right hypochondrium, tension of the muscles of the anterior abdominal wall. Positive symptom of Grekov-Ortner. What is the most likely diagnosis?

Правобічна ниркова колька Right-sided renal colic

Гострий апендицит Acute appendicitis

Гострий холецистит Acute cholecystitis

Гострий панкреатит Acute pancreatitis

Дивертикуліт Diverticulitis

3 / 200
Хвора 67-ми років скаржиться на інтенсивний біль у грудній клітці, різку задуху, серцебиття. В анамнезі -варикозне розширення вен нижніх кінцівок. Об’єктивно: шкіра ціаноти-чна, вкрита холодним потом, тахіпное (ЧД- 36/хв.), ЧСС- 120/хв., АТ- 110/70 мм рт.ст. ЕКГ: P 'pulmonаle', відхилення електричної осі серця вправо, Qііі, Sі, негативний T в ііі, V1 — V3. Який препарат найбільш доцільний у даному випадку? A 67-year-old patient complains of intense chest pain, sudden shortness of breath, palpitations. In the anamnesis - varicose veins of the lower extremities. Objectively: cyanotic skin -chne, covered with cold sweat, tachypnea (HR- 36/min.), heart rate- 120/min., BP- 110/70 mm Hg ECG: P 'pulmonale', deviation of the electrical axis of the heart to the right, Qiii, Si , negative T in iii, V1 — V3. Which drug is the most appropriate in this case?

Верапаміл Verapamil

Атенолол Atenolol

Стрептокіназа Streptokinase

Баралгін Baralgin

Но-шпа No-shpa

4 / 200
При огляді дитини 2-х місяців виявлено асиметрію складок сідниць, порушення відведення в кульшових суглобах, незначне вкорочення лівої ноги. Про що можна думати? During the examination of a 2-month-old child, asymmetry of the folds of the buttocks, impaired abduction in the hip joints, slight shortening of the left leg were found. What can we think about?

Вроджений вивих стегна Congenital hip dislocation

Варусна деформація шийки стегна Varus deformity of the femoral neck

Дитячий церебральний параліч Cerebral palsy

Запалення кульшового суглоба Inflammation of the hip joint

Травматичний вивих стегна Traumatic hip dislocation

5 / 200
Хворий 63-х років, в анамнезі 8 років ІХС, 4 роки тому Q-інфаркт міокарда. Раптово, після емоційного навантаження, виник напад серцебиття, знепритомнів. Об’єктивно: стан важкий, шкіра волога, бліда. Пульс та ЧСС не підраховуються. АТ- 130/80 мм рт.ст. ЕКГ: ЧСС- 200/хв., QRS деформовані, розширені (0,14 с), дисоціація роботи передсердь та шлуночків. Який із засобів є препаратом першої черги у даного хворого? A 63-year-old patient with an 8-year history of coronary artery disease, a Q-myocardial infarction 4 years ago. Suddenly, after emotional stress, an attack of palpitations occurred, he fainted. About objectively: the condition is severe, the skin is moist, pale. The pulse and heart rate are not counted. BP - 130/80 mm Hg. ECG: heart rate - 200/min., QRS are deformed, widened (0.14 s), dissociation of the work of the atria and ventricles. Which of the remedies is the first-line drug for this patient?

Лідокаїн Lidocaine

Натрію аденозинтрифосфат Sodium adenosine triphosphate

Строфантин Strophantin

Верапаміл Verapamil

Дигоксин Digoxin

6 / 200
У хворого 27-ми років захворювання почалося гостро: to- 39,5oC, з’явились головний біль, висип на руках, ногах, животі, який швидко поширювався впродовж доби. Об’єктивно: шкіра бліда, суха; на кінцівках, обличчі, тулубі - рясний геморагічний висип різного розміру з некрозом в центрі, крововиливи на перехідній складці кон’юнктиви. Ps- 110/хв. Печінка біля краю реберної дуги. Про яке захворювання слід думати? In a 27-year-old patient, the disease began acutely: to- 39.5oC, a headache appeared, a rash on the arms, legs, abdomen, which quickly spread during the day. Objectively: the skin is pale, dry; on the limbs, face, trunk - abundant hemorrhagic rash of various sizes with necrosis in the center, hemorrhages on the transitional fold of the conjunctiva. Ps- 110/min. Liver near the edge of the costal arch. About what disease should be considered?

Ієрсиніоз Yersiniosis

Грип Flu

Хвороба Верльгофа Werlhof's disease

Менінгококова інфекція, менінго-кокцемія Meningococcal infection, meningo-coccemia

Хвороба Шенлейн-Іеноха Schönlein-Enoch disease

7 / 200
У хворого 44-х років, що лікується з приводу гострого абсцесу середньої частки правої легені виник біль у правій половині грудної клітки, задишка, стан різко погіршився. Об’єктивно: ЧД- 28/хв., ціаноз, асиметрія грудної клітки. Над правою легенею визначається притуплення перкуторного звуку в нижніх відділах та тимпаніт в верхніх відділах. Аускультативно: різко ослаблене везикулярне дихання. Яке найбільш імовірне ускладнення виникло у хворого? A 44-year-old patient being treated for an acute abscess of the middle lobe of the right lung developed pain in the right half of the chest, shortness of breath, and his condition worsened sharply. Ob' objectively: BH- 28/min., cyanosis, asymmetry of the chest. Dullness of percussion sound in the lower parts and tympanitis in the upper parts is determined over the right lung. Auscultatively: sharply weakened vesicular breathing. What is the most probable complication that the patient has developed?

Правобічний гемопневмоторакс Right-sided hemopneumothorax

Правобічний обмежений піопнев-моторакс Right-sided limited pyopneumothorax

Правобічний тотальний піопневмо-торакс Right-sided total pyopneumothorax

Правобічний ексудативний плеврит Right-sided exudative pleurisy

Правобічна емпієма плеври Right-sided pleural empyema

8 / 200
У хворого 34-х років з протезованим мітральним клапаном, що постійно приймав синкумар, з’явився геморагічний синдром: носові кровотечі, чисельні крововиливи на шкірі, макрогематурія. В протромбіновому та аутокоагуляційних тестах майже повне незгортання крові. Які невідкладні засоби показані хворому? A 34-year-old patient with a prosthetic mitral valve who was constantly taking syncumar developed a hemorrhagic syndrome: nosebleeds, numerous skin hemorrhages, macrohematuria. prothrombin and autocoagulation tests show almost complete non-coagulability of blood. What emergency measures are indicated for the patient?

Переливання свіжозамороженої плазми Transfusion of fresh frozen plasma

Преднізолон Prednisone

Тромбоцитарна маса Platelet mass

Переливання еритроцитарної маси Red blood cell transfusion

Хлористий кальцій Calcium Chloride

9 / 200
Хвора 56-ти років страждає на ішемічну хворобу серця, що супроводжується миготливою аритмією. Вранці відчула погіршення загального стану, посилення аритмії. Під час прибирання кімнати з’явився сильний головний біль і на 15-20 хвилин хвора знепритомніла. Після опритомнення не може розмовляти й здійснювати активні рухи в правих кінцівках. Вкажіть найбільш імовірний попередній діагноз: A 56-year-old patient suffers from coronary heart disease accompanied by atrial fibrillation. In the morning, she felt a worsening of her general condition, an increase in arrhythmia. While cleaning the room, a strong headache and the patient fainted for 15-20 minutes. After regaining consciousness, she cannot speak and make active movements in her right limbs. Specify the most likely preliminary diagnosis:

Субарахноїдальний крововилив Subarachnoid hemorrhage

Геморагічний інсульт Hemorrhagic stroke

Субдуральна гематома Subdural hematoma

Тромбоемболія мозкових судин Thromboembolism of cerebral vessels

Ішемічний інсульт, обумовлений атеросклерозом мозкових судин Ischemic stroke caused by atherosclerosis of cerebral vessels

10 / 200
Хвора 40-ка років була вжалена бджолою у верхню губу. Через кілька хвилин відчула брак повітря, губа та язик сильно розпухли, з’явилася охриплість при диханні, яку стало чути на відстані. Хвора привезена рідними до міської лікарні. Об’єктивно: наявна дихальна недостатність, спричинена набряком легенів. Яка форма анафілактичного шоку має місце у даному випадку? A 40-year-old patient was stung by a bee in her upper lip. After a few minutes, she felt a lack of air, her lip and tongue became very swollen, hoarseness appeared during breathing, which could be heard in the distance. The patient was brought by relatives to the city hospital. Objectively: there is respiratory failure caused by pulmonary edema. What form of anaphylactic shock occurs in this case?

Типова Typical

Асфіктична Asphyxic

Абдомінальна Abdominal

Церебральна Cerebral

Іемодинамічна Iemodynamic

11 / 200
До приймального відділення надійшов хворий 34-х років з різким запахом алкоголю із рота. Скаржиться на головний біль, нудоту, блювання з жовчю. 6 годин тому випив у гаражі рідину, яку вважав горілкою. Об’єктивно: АТ- 200/100 мм рт.ст. Ознаки жовтяниці. При катетеризації сечового міхура сечі немає. Отруєння якою речовиною найбільш імовірне? A 34-year-old patient came to the reception department with a strong smell of alcohol from his mouth. He complains of a headache, nausea, vomiting with bile. He drank in the garage 6 hours ago a liquid that he thought was vodka. Objectively: blood pressure - 200/100 mm Hg. Signs of jaundice. There is no urine when catheterizing the bladder. Poisoning by what substance is most likely?

Етиленгліколь Ethylene glycol

Чотирихлористий вуглець Carbon tetrachloride

Оцтова есенція Vinegar Essence

Бензин Gasoline

Алкоголь Alcohol

12 / 200
До приймального відділення надійшла хвора 35-ти років, що 6 годин тому випила 100 мл столового оцту з суїцидальними намірами. Скаржиться на слабкість, спрагу, біль у роті та епі-гастрії, слинотечу. Слизова оболонка рота гіперемована. Для надання невідкладної допомоги треба: A 35-year-old patient who drank 100 ml of table vinegar 6 hours ago with suicidal intentions came to the admission department. She complains of weakness, thirst, pain in the mouth and epigastria, drooling. The mucous membrane of the mouth is hyperemic. To provide emergency care, you need:

Промити шлунок розчином калію перманганату Wash the stomach with a solution of potassium permanganate

Викликати штучне блювання Induce artificial vomiting

Не промивати шлунок Do not wash the stomach

Промити шлунок розчином соди через зонд Wash the stomach with a soda solution through a tube

Промити шлунок холодною водою через зонд Rinse the stomach with cold water through the tube

13 / 200
Робітник ферми 44-х років був госпіталізований до інфекційної лікарні зі скаргами на підвищення температури тіла до 38 — 39oC, головний біль, кволість. Хворіє третій день. Об’єктивно: на правому передпліччі - карбункул, навколо якого поширений набряк м’яких тканин; безболісний. Шкіра в зоні набряку бліда. Регіонар-ні лімфовузли збільшені, помірно болісні під час пальпації. Ps- 110/хв, АТ-100/60 мм рт.ст. Який найбільш вірогідний діагноз? A 44-year-old farm worker was hospitalized in an infectious disease hospital with complaints of an increase in body temperature to 38-39oC, headache, weakness. He has been ill for the third day. About' objectively: on the right forearm - a carbuncle, around which soft tissue swelling is widespread; painless. The skin in the area of ​​swelling is pale. Regional lymph nodes are enlarged, moderately painful during palpation. Ps- 110/min, BP-100/60 mm Hg What is the most likely diagnosis?

Сибірка Anthrax

Туляремія Tularemia

Еризипелоїд Erysipeloid

Карбункул стафілококової етіології Carbuncle of staphylococcal etiology

Бешиха Beshikha

14 / 200
У хворої 54-х років раптовий напад задишки, інтенсивний біль у грудній клітці, страх смерті Страждає на мі-тральну ваду серця. Об’єктивно: бліда з попелястим ціанозом, вибухання шийних вен; виражена тахікардія, миготлива аритмія, ЧСС- 118/хв; АТ-90/40 мм рт.ст., ЧД- 32/хв. Печінково-яремний рефлекс (симптом Плеша). ЕКГ: синдром Sі — Qііі, збільшення Pііі, зміщення сегмента STііі. Який найбільш імовірний діагноз? A 54-year-old patient has a sudden attack of shortness of breath, intense chest pain, fear of death. Suffers from mitral valve disease. Objectively: pale with ashen cyanosis, bulging neck veins, marked tachycardia, atrial fibrillation, heart rate - 118/min; blood pressure - 90/40 mmHg, blood pressure - 32/min. Hepatic-jugular reflex (Plesh's symptom). ECG: Si — Qiii syndrome , increase in Piii, displacement of the STiii segment. What is the most likely diagnosis?

Тромбоемболія легеневої артерії Thromboembolism of the pulmonary artery

Інфаркт міокарда Myocardial infarction

Серцева астма Cardiac Asthma

Пароксизми миготливої аритмії Paroxysms of atrial fibrillation

Гостра лівошлуночкова недостатність Acute left ventricular failure

15 / 200
В стаціонар доставлена хвора 30-ти років зі скаргами на болі внизу живота, затримку місячних на 3 тижні, короткочасну втрату свідомості, мізерні кров’яні виділення зі статевих шляхів. Тест на хоріонічний гонадо-тропін позитивний. Який з методів діагностики буде найбільш інформативним? A 30-year-old patient was brought to the hospital with complaints of pain in the lower abdomen, delay of menstruation for 3 weeks, short-term loss of consciousness, scanty blood discharge from the genital tract . The chorionic gonadotropin test is positive. Which of the diagnostic methods will be the most informative?

УЗД органів малого тазу Ultrasound of pelvic organs

Лапароскопія Laparoscopy

Біопсія ендометрію Endometrial biopsy

Пункція через заднє склепіння піхви Puncture through the posterior vault of the vagina

Зондування маточної порожнини Probing of the uterine cavity

16 / 200
У хворого 76-ти років розвинувся гострий великовогнищевий інфаркт міокарда з набряком легень. На фоні введення нітрогліцерину артеріальний тиск знизився до 80/40 мм рт.ст. Який з перелічених препаратів необхідно терміново ввести? A 76-year-old patient developed an acute large-focal myocardial infarction with pulmonary edema. Against the background of nitroglycerin administration, blood pressure decreased to 80/40 mm Hg. do the listed drugs need to be administered urgently?

Еуфілін Euphilin

Пентамін Pentamine

Обзидан Obzydan

Допамін Dopamine

Строфантин Strophantin

17 / 200
Хворий, який прийняв всередину 2 грами фенобарбіталу, увійшов у коматозний стан. Об’єктивно: більшість рефлексів збережені, дихання самостійне, адекватне, показники гемоди-наміки стабільні. Які лікувальні заходи слід провести? A patient who ingested 2 grams of phenobarbital entered a comatose state. Objectively: most reflexes are preserved, breathing is independent, adequate, hemodynamic indicators are stable. What medical measures should be taken?

Введення аналептиків Entering analeptics

Введення вазопресорів Introduction of vasopressors

Осмотичний форсований лужний діурез Osmotic forced alkaline diuresis

Плазмаферез Plasmapheresis

Штучна вентиляція легень Artificial lung ventilation

18 / 200
Хлопчик, що хворіє на цукровий діабет впродовж 10-ти років, надійшов до клініки через швидке погіршення стану: загальна слабкість, поліурія, полідипсія, нудота, блювання, загальмованість, сонливість. Відзначається дихання Кусмауля, запах ацетону у видихуваному повітрі. У сечі відзначається помірна кількість цукру і ацетонових тіл. Яке ускладнення цукрового діабету у даного хворого? A boy suffering from diabetes for 10 years came to the clinic due to rapid deterioration of his condition: general weakness, polyuria, polydipsia, nausea, vomiting, retardation , drowsiness. Kussmaul breathing is noted, the smell of acetone in exhaled air. A moderate amount of sugar and acetone bodies is noted in the urine. What is the complication of diabetes in this patient?

Гіперглікемічна кома Hyperglycemic coma

Кетоацидотична кома Ketoacidotic coma

Лактацидотична кома Lactacidotic coma

Печінкова кома Hepatic coma

Гіпоглікемічна кома Hypoglycemic coma

19 / 200
Для купірування головного болю дитина прийняла per os 1 таблетку анальгіну. Через деякий час у неї з’явилися і стали прогресивно наростати набряклість обличчя, повік, вух, утруднене дихання, осиплість голосу. Попередній діагноз: набряк Квінке. З якого препарату доцільно починати терапію? To relieve a headache, the child took 1 tablet of Analgin per os. After some time, swelling of the face, eyelids, ears, difficulty breathing appeared and began to grow progressively , hoarseness of voice. Previous diagnosis: Quincke's edema. With which drug is it advisable to start therapy?

Адреналін Adrenaline

Еуфілін Euphilin

Гідрокарбонат натрію Sodium bicarbonate

Димедрол Diphenhydramine

Преднізолон Prednisone

20 / 200
Вагітна 38-ми років у терміні вагітності 33 тижні, звернулася зі скаргами на головний біль, мерехтіння 'му-шок'перед очима, шум у вухах, біль у епігастральній ділянці, нудоту, блювання. Об’єктивно: ожиріння ІІ ступеня, набряки гомілок, передньої черевної стінки, АТ-160/110 мм рт.ст. У сечі - білок 1,5 г/л. Який найбільш імовірний діагноз? A 38-year-old pregnant woman, 33 weeks pregnant, complained of a headache, 'mu-shock' flickering before the eyes, tinnitus, pain in in the epigastric area, nausea, vomiting. Objectively: obesity of the II degree, swelling of the lower legs, anterior abdominal wall, BP-160/110 mm Hg. Protein in urine - 1.5 g/l. What is the most likely diagnosis?

Еклампсія Eclampsia

Гостре порушення мозкового кровообігу Acute cerebrovascular accident

Гіпертонічний криз Hypertensive crisis

Прееклампсія легкого ступеня Mild preeclampsia

Прееклампсія середнього ступеня Moderate preeclampsia

21 / 200
Вночі у хворого з’явилися задуха, кашель з рясним харкотинням рожевого кольору. З анамнезу: страждає на гіпертонічну хворобу, варикозне розширення вен обох кінцівок, 2 роки тому переніс інфаркт міокарда. Об’єктивно: неспокійний, положення ор-топное, акроціаноз, ЧДР- 40/хв. АТ-220/110 мм рт.ст. У легенях - різнокаліберні середньо- і великопухирчасті хрипи. Який найбільш імовірний діагноз? At night, the patient developed shortness of breath, a cough with copious sputum of pink color. From the anamnesis: he suffers from hypertension, varicose veins of both extremities, 2 years ago he suffered myocardial infarction. Objectively: restless, orthotopic position, acrocyanosis, CHDR-40/min. BP-220/110 mm Hg. In the lungs - medium- and large-caliber rales. What is the most likely diagnosis?

Легенева кровотеча Pulmonary hemorrhage

Крупозна пневмонія Croup pneumonia

Тромбоемболія легеневої артерії Thromboembolism of the pulmonary artery

Гостра лівошлуночкова недостатність Acute left ventricular failure

Астматичний статус Asthmatic status

22 / 200
У хворого з отруєнням опіатами має місце різке брадипное, виражений ціаноз. Яка першочергова дія лікаря? A patient with opiate poisoning has acute bradypnea, pronounced cyanosis. What is the doctor's first action?

Проведення штучної вентиляції легень Carrying out artificial lung ventilation

Киснетерапія Oxygen therapy

Введення еуфіліну Introducing Euphilin

Введення налоксону Naloxone administration

Введення атропіну Atropine administration

23 / 200
До стаціонару надійшов хворий, який близько години тому помилково випив близько 150 мл технічного спирту, припустимо метилового. Інших скарг не висловлює. Об’єктивно: гемодинаміка стабільна, зовнішнє дихання не порушене, діурез достатній. Який антидот необхідно застосувати? A patient was admitted to the hospital who, about an hour ago, mistakenly drank about 150 ml of technical alcohol, let's assume methyl alcohol. He has no other complaints. Objectively: hemodynamics are stable, external breathing is not impaired, diuresis is sufficient. What antidote should be used?

Дефероксамін Deferoxamine

Етанол Ethanol

Прозерин Prozerin

Атропіну сульфат Atropine sulfate

Налоксон Naloxone

24 / 200
Хворий під час проведення внутрішньовенної урографії з використанням уротрасту знепритомнів. Об’єктивно: ціаноз шкірних покривів, брадипное, ЧД- 5-6/хв. Артеріальний тиск не визначається, пульс на сонних артеріях з частотою 130-140/хв. Першочерговим заходом буде: During intravenous urography using urotrast, the patient fainted. Objectively: cyanosis of the skin, bradypnea, heart rate - 5-6/min. Blood pressure is not determined , pulse on the carotid arteries with a frequency of 130-140/min. The primary measure will be:

В/в введення великих доз глюко-кортикоїдів IV administration of large doses of glucocorticoids

Проведення ШВЛ Ventilation

Непрямий масаж серця Indirect heart massage

Введення атропіну в/в Introduction of atropine IV

Введення адреналіну в/в Adrenaline injection IV

25 / 200
Хворий 42-х років після прийому алкоголю заснув навпочіпки і проспав близько 12 годин. Через деякий час після пробудження з’явилися парестезії, набряклість і болісність у ділянці гомілок, болі у поперековій ділянці. На другу добу після випадку діурез знизився до 150 мл/добу, сеча лаково-червоного кольору. АТ-140/90 мм рт.ст. ЧСС- 100/хв. Чим зумовлені такі симптоми? A 42-year-old patient fell asleep on his back after drinking alcohol and slept for about 12 hours. Some time after waking up, paresthesias, swelling and pain appeared in the lower legs, pains in the lumbar region. On the second day after the incident, diuresis decreased to 150 ml/day, urine was varnish-red in color. BP-140/90 mmHg, heart rate-100/min. What are the causes of such symptoms?

Отруєння етиленгліколем Ethylene glycol poisoning

Гострий гломерулонефрит Acute glomerulonephritis

Отруєння метиловим спиртом Methyl alcohol poisoning

Хронічний алкоголізм Chronic alcoholism

Міоглобінурійний тубулонекроз Myoglobinuric tubulonecrosis

26 / 200
Хвора 37-ми років доставлена в інфекційний стаціонар у непритомному стані на 5-й день хвороби з вогнища кору. На шкірі тулуба, верхніх і нижніх кінцівок велико-плямистий висип з елементами лущення. Лімфаде-нопатія, помірний гепатолієнальний синдром, тахіпное, тахікардія. Менін-геальні симптоми: Керніга позитивний і ригідність м’язів шиї. Виявлено правобічний геміпарез, позитивні симптоми Бабінського, Опенгейма, Гордона. Яке ускладнення найбільш імовірне? A 37-year-old patient was brought to an infectious disease hospital in an unconscious state on the 5th day of illness from a measles outbreak. On the skin of the trunk, upper and lower extremities, a large spotted a rash with elements of peeling. Lymphadenopathy, moderate hepatolienal syndrome, tachypnea, tachycardia. Meningeal symptoms: Kernig positive and neck muscle stiffness. Right-sided hemiparesis, positive symptoms of Babinski, Oppenheim, Gordon. What complication is most likely?

Менінгококовий сепсис Meningococcal sepsis

ГПМК за геморагічним типом Hemorrhagic hemorrhagic type

Менінгококовий менінгіт Meningococcal meningitis

Менінгоенцефаліт Meningoencephalitis

ГПМК за ішемічним типом Ischemic type of HPMC

27 / 200
До лікаря звернулася хвора 25-ти років зі скаргами на болі внизу живота, підвищену температуру до 38oC, гноєподібні виділення із статевих органів. Хворіє впродовж 3-х днів. Захворювання пов’язує з перенесеною операцією аборту терміном 8-9 тижнів. Об’єктивно: температура тіла 38oC, Ps- 86/хв., АТ- 120/80 мм рт.ст. Живіт м’який, болючий під час пальпації над лобком. Виділення сукровично-гнійні. Який найбільш імовірний діагноз? A 25-year-old patient came to the doctor with complaints of pain in the lower abdomen, elevated temperature up to 38oC, purulent discharge from the genitals. She has been ill for 3 days. The disease is associated with a postponed abortion operation for 8-9 weeks. Objectively: body temperature 38oC, Ps- 86/min., BP- 120/80 mm Hg. The abdomen is soft, painful during palpation above the pubis. Sucre-purulent discharge. What is the most likely diagnosis?

Кольпіт Colpit

Вульвовагініт Vulvovaginitis

Цистит Cystitis

Аднексит Adnexitis

Ендометрит Endometritis

28 / 200
У чоловіка 53-х років, хворого на цироз печінки алкогольного генезу, після ОРВІ, з’явились збудженість, дезорієнтованість, галюцинації, підвищення температури тіла до 38oC, значна жовтяниця, запах 'дохлої ми-ші'з рота, тахікардія. У крові: Kb- 92 г/л, лейк.- 11 • 109/л, ШОЕ- 32 мм/год, АлАТ- 4,56 мкмоль/л, АсАТ- 2,68 мкмоль/л, білірубін загальний - 180 мкмоль/л, непрямий - 60 мкмоль/л, ПТІ- 58%. Який діагноз найбільш імовірний? A 53-year-old man with cirrhosis of the liver of alcoholic origin, after SARS, developed excitement, disorientation, hallucinations, an increase in body temperature up to 38oC, significant jaundice, the smell of 'dead mouse' from the mouth, tachycardia. In the blood: Kb- 92 g/l, leuk.- 11 • 109/l, ESR- 32 mm/h, AlAT- 4.56 μmol/l, AsAT - 2.68 μmol/l, total bilirubin - 180 μmol/l, indirect - 60 μmol/l, PTI - 58%. What is the most likely diagnosis?

Алкогольний делірій Alcoholic delirium

Гостра ниркова недостатність Acute renal failure

Прогресуюча печінкова недостатність Progressive liver failure

Гострий гепатит Acute hepatitis

Гіперкетонемічна діабетична кома Hyperketonemic diabetic coma

29 / 200
Хвора 74-х років хворіє на цукровий діабет другого типу, приймає ма-нініл. Протягом двох тижнів приймає по 2 таблетки гіпотіазиду. Об’єктивно: свідомість сплутана, шкірні покриви сухі, теплі, тургор м’язів знижений, температура тіла - 38oC, Ps- 120/хв., АТ60/40 мм рт.ст., дихання поверхневе, язик сухий, запах ацетону відсутній. Глюкоза крові 34 ммоль/л, ацетон в сечі - слабкопозитивний, Nа+ сироватки - 162 ммоль/л, K+ - 3, 0 ммоль/л. Який розчин показаний для застосування в першу чергу? A 74-year-old patient suffers from type 2 diabetes, takes maninil. She takes 2 hypothiazide tablets for two weeks. Objectively: consciousness is confused, skin is dry, warm, muscle turgor is reduced, body temperature - 38oC, Ps- 120/min., blood pressure 60/40 mm Hg, shallow breathing, dry tongue, no smell of acetone, blood glucose 34 mmol/l, acetone in urine - weakly positive, serum Na+ - 162 mmol/l, K+ - 3.0 mmol/l. What solution is indicated for use in the first place?

Поляризуюча суміш Polarizing mixture

Розчин Рінгера Ringer's solution

5% розчин глюкози 5% glucose solution

Фізіологічний розчин NаCl NaCl physiological solution

Гіпотонічний розчин NаCl Hypotonic NaCl solution

30 / 200
Дитина 5-ти років, яка випила невідому рідину, скаржиться на біль за грудниною та в епігастральній ділянці. Було багаторазове блювання шлунковим вмістом, слизом. На губах та в роті - гіперемія та набряк слизової оболонки, поодинокі ерозії, білуваті плями. Лікар запідозрив хімічний опік стравоходу. В чому буде полягати оптимальна невідкладна допомога? A 5-year-old child, who drank an unknown liquid, complains of pain behind the sternum and in the epigastric area. There was repeated vomiting of stomach contents, mucus. On the lips and in mouth - hyperemia and swelling of the mucous membrane, isolated erosions, whitish spots. The doctor suspected a chemical burn of the esophagus. What would be the optimal emergency care?

Беззондове промивання шлунка Tubeless gastric lavage

Зондове промивання шлунка, дезін-токсикаційна терапія Gastric probe lavage, detoxification therapy

Обробка слизової оболонки рота дезінфікуючими засобами Treatment of oral mucosa with disinfectants

Інфузійна терапія Infusion therapy

Рясне лужне пиття Abundant alkaline drinking

31 / 200
У хворого 54-х років після стресу з’явились різкий біль у ділянці серця, запаморочення, задишка, відчуття страху. Об’єктивно: периферичний ціаноз, шкіра холодна, липка. ЧД21 /хв, Ps- 120/хв, АТ- 80/60 мм рт.ст. При фізикальному обстеженні виявлено ритм галопу, розповсюджені застійні хрипи в легенях. На ЕКГ: значне зниження амплітуди зубця R в і, аvL, V\-6 відведеннях з елевацією сегменту ST. Призначення якого препарату є найбільш доцільним в даному випадку? A 54-year-old patient developed sharp pain in the heart area after stress, dizziness, shortness of breath, a feeling of fear. Objectively: peripheral cyanosis, cold skin , sticky. Blood pressure 21/min, Ps- 120/min, BP- 80/60 mm Hg. Physical examination revealed a gallop rhythm, widespread congestive rales in the lungs. On the ECG: a significant decrease in the amplitude of the R wave in i, avL, V \-6 leads with ST segment elevation. Which drug is the most appropriate to prescribe in this case?

Допамін Dopamine

Добутамін Dobutamine

Серцеві глікозиди Cardiac glycosides

Левартеренол Levarterenol

Ізопротеренол Isoproterenol

32 / 200
Хворий 50-ти років викликав бригаду швидкої допомоги. Скаржиться на напади болю в правому боці, при якому не може вибрати позицію. Біль віддає в пахвинну ділянку. При сечовипусканні після нападу відмічав різь та червоне забарвлення сечі. Подібний біль вже турбував хворого, але пройшов після прийняття гарячої ванни. Об’єктивно: стан середньої важкості, збуджений. Хворий міцної статури, підвищеного харчування, Ps78 /хв., АТ- 140/80 мм рт.ст., ЧД- 16/мин., to- 37,2oC. Живіт м’який, безболісний, симптом Пастернацько-го різко позитивний справа. Який діагноз найбільш імовірний? A 50-year-old patient called an ambulance. He complains of attacks of pain in his right side, in which he cannot choose a position. The pain radiates to the groin area. When urinating after the attack, he noted cutting and red coloration of the urine. Similar pain had already bothered the patient, but it passed after taking a hot bath. Objectively: the condition is of medium severity, excited. The patient is of a strong physique, with increased nutrition, Ps78/min., BP- 140/80 mm Hg, blood pressure - 16/min, to - 37.2oC. Abdomen is soft, painless, Pasternatsky's symptom is sharply positive. What is the most likely diagnosis?

Оперізуючий лишай Zingles

Кишкова колька Intestinal colic

Ниркова колька Renal colic

Печінкова колька Hepatic colic

Гостра невралгія Acute neuralgia

33 / 200
Дитина 5-ти років захворіла раптово: підвищилася температура тіла до 39,7oC, на стегнах та сідницях з’явилася геморагічна висипка зірчастої форми розміром від 0,5 до 3 см, акроціа-ноз, кінцівки холодні, ниткоподібний пульс. Лікар швидкої допомоги запідозрив менінгококцемію. Який антибіотик необхідно ввести дитині на до-госпітальному етапі? A 5-year-old child fell ill suddenly: the body temperature rose to 39.7oC, a star-shaped hemorrhagic rash appeared on the hips and buttocks, the size of 0.5 to 3 cm, acrocyanosis, cold extremities, thread-like pulse. The emergency physician suspected meningococcemia. What antibiotic should be administered to the child at the pre-hospital stage?

Оксацилін Oxacillin

Цефазолін Cefazolin

Левоміцетину сукцинат Levomycetin succinate

Рифампіцин Rifampicin

Іентаміцин Ientamycin

34 / 200
У дитини 5-ти років на фоні проявів ГРВ! з’явилися блювання до 4 разів на добу, зниження апетиту, запах ацетону з рота. В сечі - наявність ацетону (++++). Який найбільш імовірний діагноз? A 5-year-old child had vomiting up to 4 times a day, decreased appetite, the smell of acetone from the mouth. In the urine - the presence acetone (++++). What is the most likely diagnosis?

Синдром ацетонемічного блювання Acetonemic vomiting syndrome

Іострий панкреатит Acute pancreatitis

Шлунково-кишкова кровотеча Gastrointestinal bleeding

Гостра інфекція сечовивідних шляхів Acute urinary tract infection

Іострий гастрит Acute gastritis

35 / 200
Хворий 44-х років скаржиться на 'голодні'нічні болі в епігастрії, загальну слабкість, нездужання, печію, запаморочення. Двічі були дьог-теподібні випорожнення. Об’єктивно: шкірні покриви бліді. Ps- 98/хв., АТ- 90/60 мм рт.ст. Живіт дещо болісний у епігастрії. При пальцевому дослідженні - мелена. У крові: ер.-2,8 • 1012/л, НЬ- 64 г/л. Який найбільш імовірний діагноз? A 44-year-old patient complains of 'hunger' night pains in the epigastrium, general weakness, malaise, heartburn, dizziness. Twice there were tar-like stools. About' objectively: the skin is pale. Ps- 98/min., BP- 90/60 mm Hg. The abdomen is somewhat painful in the epigastrium. On finger examination - melena. In the blood: er.-2.8 • 1012/l, Hb - 64 g/l. What is the most likely diagnosis?

Виразкова хвороба 12-палої кишки, ускладнена кровотечею Duodenal ulcer complicated by bleeding

Гострий кровоточивий геморой Acute bleeding hemorrhoids

Неспецифічний виразковий коліт Nonspecific ulcerative colitis

Синдром Мелорі-Вейса Mallory-Weiss Syndrome

Кровотеча з варикозно розширених вен стравоходу Bleeding from varicose veins of the esophagus

36 / 200
Хворий 60-ти років скаржиться на біль, набряк, ціаноз правої руки, починаючи з верхньої третини плеча. Напередодні вживав спиртні напої у великій кількості. Об’єктивно: стан середньої важкості, праве плече на 8 см товстіше за ліве, шкірні покриви правої руки ціанотичні, набряклі; пульс на променевій артерії ослаблений, рухи пальців обмежені. Сила м’язів знижена. Температура тіла 37,8oC. Який найбільш вірогідний діагноз? A 60-year-old patient complains of pain, swelling, cyanosis of the right arm, starting from the upper third of the shoulder. The day before, he drank a lot of alcohol. Objectively: the condition is of moderate severity, the right shoulder is 8 cm thicker than the left, the skin of the right hand is cyanotic, swollen; the pulse on the radial artery is weakened, the movements of the fingers are limited. Muscle strength is reduced. Body temperature is 37.8oC. What is the most likely diagnosis?

Тромбоз правої плечової вени Thrombosis of the right brachial vein

Емболія плечової артерії Brachial embolism

Атеросклеротичне ураження плечової артерії, стеноз Atherosclerotic lesion of the brachial artery, stenosis

- -

Неврит променевого нерва Radial neuritis

37 / 200
Постраждалий 42-х років, виявлений на місці ДТП, скаржиться на біль у ділянці таза та правого стегна. Об’єктивно: загальмований, стогне, шкіра бліда, язик сухий. ЧДР- 28/хв. Праве стегно деформоване, набрякле, у верхній його третині патологічна рухливість, крепітація кісткових уламків. Великий крововилив, набряк та біль під час пальпації в правих пахвинній, здухвинній та сідничній ділянках. Ps- і12/хв, АТ- 80/45 мм рт.ст. Якому ступеню травматичного шоку відповідає стан потерпілого? A 42-year-old victim found at the scene of an accident complains of pain in the pelvis and right thigh. Objectively: slowed down, moaning, pale skin, tongue dry. ChDR- 28/min. The right thigh is deformed, swollen, in its upper third there is pathological mobility, crepitation of bone fragments. Large hemorrhage, swelling and pain during palpation in the right inguinal, pubic and buttock areas. Ps- and 12/min, BP - 80/45 mm Hg What degree of traumatic shock does the victim's condition correspond to?

- -

ііі iii

і and

іV иV

іі ii

38 / 200
Дитина 1-го року надійшла до інфекційного відділення зі скаргами на багаторазове невпинне блювання, профузні рідкі випорожнення. Об’єктивно: адинамічна, риси обличчя загострені, тургор тканин різко знижений, акроціаноз. Шкірні покриви бліді, холодні з 'мармуро-вим'малюнком. Дихання поверхневе, тони серця глухі, почащені. АТ- 55/35 мм рт.ст. Анурія. Який невідкладний стан у дитини? A 1st-year-old child was admitted to the infectious department with complaints of repeated incessant vomiting, profuse liquid stools. Objectively: adynamic, facial features are sharp, tissue turgor is sharp decreased, acrocyanosis. The skin is pale, cold with a 'marble-like' pattern. Breathing is shallow, heart sounds are dull, increased. Blood pressure - 55/35 mm Hg. Anuria. What is the emergency condition of the child?

Гостра надниркова недостатність Acute adrenal insufficiency

Гіповолемічний шок Hypovolemic shock

Гостра судинна недостатність Acute vascular insufficiency

Гостра ниркова недостатність Acute renal failure

Гостра серцева недостатність Acute heart failure

39 / 200
У дівчини 13-ти років при тривалому перебуванні в задушливому приміщенні з’явилися скарги на запаморочення, слабкість. Різко зблідла, тіло вкрилося холодним потом, шкіра набула 'мармурового'малюнку, з’явилися ціаноз губ, сопорозна свідомість, часте поверхневе дихання, тахікардія. Тони серця послаблені, АТ- 50/10 мм рт.ст. Який найбільш вірогідний діагноз? A 13-year-old girl complained of dizziness and weakness after being in a stuffy room for a long time. She turned pale sharply, her body became covered in cold sweat, her skin became 'marble 'picture, cyanosis of the lips, sedated consciousness, frequent shallow breathing, tachycardia appeared. Heart tones are weakened, blood pressure - 50/10 mm Hg. What is the most likely diagnosis?

Мігрень Migraine

Гостра серцева недостатність Acute heart failure

Задишечно-ціанотичний напад Dypnoea-cyanotic attack

- -

Колапс Collapse

40 / 200
Хлопчик 8-ми років спостерігається з приводу гемофілії А, надійшов у дитяче відділення у зв’язку з травмою лівої поперекової ділянки, що відбулася 5 годин тому. Скаржиться на млявість, біль у лівому боці, домішок крові у сечі. Об’єктивно: стан важкий, блідий, млявий. У сечі: ма-крогематурія (еритроцити - усе поле зору). Яку терапію необхідно провести при даному невідкладному стані першочергово? An 8-year-old boy is being observed for hemophilia A, was admitted to the pediatric department due to an injury to the left lumbar region that occurred 5 hours ago. He complains of lethargy, pain in the left side, an admixture of blood in the urine. Objectively: the condition is heavy, pale, lethargic. In the urine: macrohematuria (erythrocytes - the entire field of vision). What therapy should be carried out in this urgent condition as a matter of priority?

В/в кріопреципітат IV cryoprecipitate

В/в еритроцитарна маса IV erythrocyte mass

Переливання одногрупної крові Transfusion of identical blood

В/в відмиті еритроцити IV washed erythrocytes

В/в альбумін IV albumin

41 / 200
Хворий 60-ти років в дуже важкому стані. Непритомний. Шкіра і склери жовтяничні. Ps130 /хв., АТ-90/40 мм рт.ст. Тони серця ослаблені. Печінковий запах з рота. Розміри печінки та селезінки збільшені. У крові: метаболічний ацидоз (рН 4,1), АсАТ- 1,6 мкмоль/лгод, АлАТ- 2,0 мкмоль/лгод; загальний білірубін -318,6 мкмоль/л, натрій - 90 ммоль/л, калій - 3,3 ммоль/л. Який попередній діагноз? A 60-year-old patient is in a very serious condition. Unconscious. Skin and sclera are jaundiced. Ps130/min., BP-90/40 mmHg. Tons the hearts are weakened. Hepatic odor from the mouth. The sizes of the liver and spleen are increased. In the blood: metabolic acidosis (pH 4.1), AsAT- 1.6 μmol/lh, AlAT- 2.0 μmol/lh; total bilirubin -318.6 μmol/l, sodium - 90 mmol/l, potassium - 3.3 mmol/l. What is the previous diagnosis?

Гіперглікемічна кома Hyperglycemic coma

Алкогольна кома Alcoholic coma

Уремічна кома Uremic coma

Гіпоглікемічна кома Hypoglycemic coma

Печінкова кома Hepatic coma

42 / 200
Хвора 32-х років надійшла з діагнозом: цукровий діабет кетоацидо-тичний стан. Після проведеного інтенсивного лікування у хворої з’явилась неадекватна поведінка, агресивність, галюцинації, знепритомніла. Об’єктивно: шкіра волога, язик вологий. Дихання рівне, спокійне, 20/хв., ЧСС- 120/хв., АТ- 90/60 мм рт.ст. Тонус м’язів підвищений. Клонічні та тонічні судоми. Вогнищевих ознак з боку центральної нервової системи немає, рівень цукру в крові 2,6 ммоль/л. Який попередній діагноз? A 32-year-old patient arrived with a diagnosis of diabetes ketoacidotic state. After intensive treatment, the patient developed inappropriate behavior, aggressiveness, hallucinations, fainted . Objectively: the skin is moist, the tongue is moist. Breathing is even, calm, 20/min., heart rate - 120/min., BP - 90/60 mm Hg. Muscle tone is increased. Clonic and tonic convulsions. Focal There are no central nervous system symptoms, the blood sugar level is 2.6 mmol/l. What is the previous diagnosis?

Молочнокисла кома Lactic coma

Порушення мозкового кровообігу Disruption of cerebral circulation

Гіперглікемічна кома Hyperglycemic coma

Кетоацидотична кома Ketoacidotic coma

Гіпоглікемічна кома Hypoglycemic coma

43 / 200
Після вживання апельсинового соку у 8-місячної дитини на шкірі тулуба, кінцівок, обличчя з’явилися пу- хирці, що виступають над поверхнею шкіри, сверблячка. Батьки звернулися до лікаря. Стан дитини задовільний. Яку допомогу необхідно надати дитині? After drinking orange juice, an 8-month-old child developed blisters protruding above the surface of the skin, itching on the skin of the body, limbs, and face. The parents contacted to the doctor. The child's condition is satisfactory. What help should be given to the child?

Не вживати апельсиновий сік Don't drink orange juice

Внутрішньом’язово антигістамінні, гормональні препарати Intramuscular antihistamines, hormonal drugs

Антигістамінні препарати Antihistamines

Елімінаційна дієта Elimination Diet

Елімінаційна дієта, ентеросорбенти Elimination diet, enterosorbents

44 / 200
Хлопчика 13-ти років госпіталізовано у відділення реанімації з діагнозом 'синдром Лайєлла'. Об’єктивно: температура 39,1oC, Ps- 128/хв., ЧД-30/хв. Яку допомогу в першу чергу необхідно надати дитині? A 13-year-old boy was hospitalized in the intensive care unit with a diagnosis of Lyell's syndrome. Objectively: temperature 39.1oC, Ps- 128/min., BH -30/min. What kind of help should be provided to the child in the first place?

Сечогінні препарати Diuretics

Глюкокортикоїдні гормони Glucocorticoid hormones

Серцеві глікозиди Cardiac glycosides

Седативні препарати Sedatives

Жарознижуючі препарати Antipyretic drugs

45 / 200
Дівчина 14-ти років на уроці фізичного виховання при різкій зміні положення тіла раптово знепритомніла. Об’єктивно: низький артеріальний тиск, рідкий слабкий пульс. Про який стан слід думати? A 14-year-old girl suddenly fainted during a physical education lesson after a sudden change in body position. Objectively: low blood pressure, rare weak pulse. What condition should be reported think?

Серцева недостатність Heart failure

Непритомність Fainting

Кардіогенний шок Cardiogenic shock

Колапс Collapse

Ортостатична гіпотензія Orthostatic hypotension

46 / 200
У дитини віком 10 місяців, на тлі перебігу гострої респіраторної вірусної інфекції, раптово з’явилась різка млявість, блідість шкірних покровів, блювання, підвищилась температура тіла до 39oC. При обстеженні виявлена глухість тонів серця, тахікардія - 160/хв., артеріальна гіпотонія - 70/50 мм рт.ст. Вміст Nа+ плазми -125 ммоль/л, Сі_ плазми - 84 ммоль/л, глюкози - 2,8 ммоль/л. З яким ускладненням пов’язано погіршення стану дитини? A 10-month-old child, against the background of an acute respiratory viral infection, suddenly developed lethargy, pale skin, vomiting, and the body temperature rose to 39oC. During the examination, deafness of heart sounds, tachycardia - 160/min., arterial hypotension - 70/50 mm Hg, Na+ plasma content - 125 mmol/l, Si_ plasma - 84 mmol/l, glucose - 2.8 mmol/l l. What complication is associated with the deterioration of the child's condition?

Гіпотонічний тип ексикозу Hypotonic type of exicosis

Гостре запалення легенів Acute pneumonia

Гіпоглікемічна кома Hypoglycemic coma

Гостра недостатність надниркових залоз Acute adrenal insufficiency

Гостра серцева недостатність Acute heart failure

47 / 200
Хвора 68-х років хворіє протягом 10-ти років на цукровий діабет 1 типу. Напередодні розвинулося погіршення стану: з’явилися загальна слабкість, млявість. Об’єктивно: шкіра суха, тургор м’яких тканин знижений. Кома 1, Ps- 102/хв., ритмічний, м’який. АТ- 90/60 мм рт.ст. Глюкоза крові - 50 ммоль/л, глюкозурія, ацетон не виявлений. Осмолярність плазми - 370 мо-смоль/л. Який стан розвинувся у хворої? A 68-year-old patient has been suffering from type 1 diabetes for 10 years. The day before, her condition worsened: general weakness, lethargy appeared. Objectively : the skin is dry, the turgor of soft tissues is reduced. Coma 1, Ps- 102/min., rhythmic, soft. BP- 90/60 mm Hg. Blood glucose - 50 mmol/l, glucosuria, acetone is not detected Plasma osmolarity is 370 mo-smol/l. What condition has developed in the patient?

Кетоацидотична кома Ketoacidotic coma

Гіперосмолярна кома Hyperosmolar coma

Гіпоглікемічна кома Hypoglycemic coma

Гіпертиреоїдна кома Hyperthyroid coma

Лактацидотична кома Lactacidotic coma

48 / 200
Хворий 32-х років з невідомим анамнезом доставлений бригадою швидкої допомоги. Об’єктивно: шкірні покриви бліді. Свідомість - кома 1, зіниці D=S, фотореакція збережена. Шкіра волога, тургор м’яких тканин не знижений. Дихання самостійне, ЧД18 /хв. Ps- 98/хв., ритмічний. АТ- 130/80 мм рт.ст., глюкоза крові - 2,5 ммоль/л. Яка лікувальна тактика? A 32-year-old patient with an unknown history was delivered by an ambulance. Objectively: the skin is pale. Consciousness - coma 1, pupils D=S, photoreaction is preserved The skin is moist, the turgor of soft tissues is not reduced. Breathing is independent, BH18/min. Ps- 98/min., rhythmic. BP- 130/80 mm Hg, blood glucose - 2.5 mmol/l. What therapeutic tactics?

Преднізолон в/в Prednisolone IV

0,9% NаCl в/в 0.9% NaCl IV

7,2% NаCl в/в 7.2% NaCl IV

40% глюкоза в/в 40% glucose IV

5% глюкоза в/в 5% glucose IV

49 / 200
Хворий 38-ми років, робітник автосервісу, доставлений до клініки в стані коми ііі ступеня. Об’єктивно: клонічні судоми, неконтрольоване сечовиділення. Розлади дихання за типом Чейн-Стокса, тахікардія, червоний колір обличчя та слизових оболонок. Якою речовиною отруївся хворий? A 38-year-old patient, a car service worker, was brought to the clinic in a coma of the iii degree. Objectively: clonic convulsions, uncontrolled urination. Cheyne-type respiratory disorders -Stokes, tachycardia, red color of the face and mucous membranes. What substance was the patient poisoned by?

Чадний газ Carbon monoxide

Пари ртуті Mercury vapors

Тетраетилсвинець Tetraethyllead

Пари дихлоретану Dichloroethane Vapors

Пари бензину Gasoline vapors

50 / 200
У хворого 35-ти років після враження блискавкою відмічалася зупинка кровообігу протягом 5-6 хвилин. Об’єктивно: кома і ст. Тонічні судоми. Зіниці вузькі, D=S, фотореакції мляві, дифузне підвищення м’язового тонусу за екстрапірамідним типом. Яка це кома? In a 35-year-old patient, after being struck by lightning, blood circulation stopped for 5-6 minutes. Objectively: coma and arteriosclerosis. Tonic convulsions. Pupils are narrow, D =S, photoreactions are sluggish, diffuse increase in muscle tone according to the extrapyramidal type. What kind of coma is this?

Епілептична Epileptic

Травматична Traumatic

Алкогольна Alcohol

Постгіпоксична Posthypoxic

Апоплектична Apoplectic

51 / 200
У дівчини 19-ти років запідозрена недостатність наднирників (хвороба Аддісона). Що з нижче перерахованого може допомогти при підтвердженні діагнозу? A 19-year-old girl is suspected of having adrenal insufficiency (Addison's disease). Which of the following can help confirm the diagnosis?

Гіпокаліємія Hypokalemia

Гіпонатріємія Hyponatremia

Артеріальна гіпертензія Hypertension

Гіперглікемія Hyperglycemia

Гіпотермія Hypothermia

52 / 200
Студент 19-ти років скаржиться на сухий кашель, дряпання в горлі, підвищення температури тіла до 38,1oC. За порадою друга приймав ампіцилін. Через 2 дні після початку захворювання з’явилася висипка по всьому тілу, рожевого кольору, 1 см у діаметрі, дещо припіднята. Який НАЙМЕНШ вірогідний діагноз? A 19-year-old student complains of a dry cough, a scratchy throat, an increase in body temperature to 38.1oC. He took ampicillin on the advice of a friend. 2 days after the start disease, a rash appeared all over the body, pink in color, 1 cm in diameter, slightly raised. What is the LEAST likely diagnosis?

- -

Менінгококцемія Meningococcemia

Мононуклеоз Mononucleosis

Кір Measles

Медикаментозний дерматит Drug dermatitis

53 / 200
Хворий 22-х років доставлений у лікарню зі скаргами на задишку, ядуху, сухий кашель. Стан погіршився 2 доби тому. За останні 12 годин користувався беротеком біля 50 разів. Об’єктивно: ЧД- 30/хв., дистанційні хрипи. Обличчя ціанотичне. У легенях вислуховуються ділянки 'німої легені'. Чим ускладнився перебіг бронхіальної астми? A 22-year-old patient was brought to the hospital with complaints of shortness of breath, wheezing, dry cough. His condition worsened 2 days ago. In the last 12 hours, he used Berotek about 50 times . Objectively: heart rate - 30/min., distant wheezing. The face is cyanotic. In the lungs, areas of the 'dumb lung' can be heard. What complicated the course of bronchial asthma?

Астматичний статус і стадії Asthmatic status and stages

Астматичний статус ііі стадії Asthmatic status of stage III

Ексудативний плеврит Exudative pleurisy

Астматичний статус іі стадії Asthmatic status of stage II

Ателектаз легень Pulmonary atelectasis

54 / 200
Хлопчик віком 1 рік 8 місяців доставлений до приймального відділення 'швидкою'. При огляді стан дуже важкий. Притомний. Менінгеаль-них симптомів немає. Температура тіла 39 oC, кінцівки холодні. Виражені розлади мікроциркуляції. На нижніх кінцівках, сідницях, тулубі, обличчі - зливна 'зірчаста'геморагічна висипка, що швидко розповсюджується. Невеличкі крововиливи на слизових оболонках. Висипка з’явилась декілька годин тому. ЧД- 54/хв. ЧСС- 180/хв. АТ- 65/25 мм рт.ст. Який найбільш імовірний діагноз? A 1-year-8-month-old boy was brought to the emergency room. On examination, the condition is very serious. He is unconscious. There are no meningeal symptoms. Body temperature is 39 oC, limbs are cold. Pronounced disorders of microcirculation. On the lower limbs, buttocks, trunk, face - a draining 'star' hemorrhagic rash that spreads quickly. Small hemorrhages on the mucous membranes. The rash appeared a few hours ago. BH- 54/min. HR- 180/min. BP - 65/25 mm Hg. What is the most likely diagnosis?

Геморагічний васкуліт Hemorrhagic vasculitis

Аддісоновий криз Addison's Crisis

Геморагічна лихоманка Hemorrhagic fever

Блискавична менінгококцемія Bulky meningococcemia

Тромбоцитопенічна пурпура Thrombocytopenic purpura

55 / 200
У дитини через 4 години після проведення спинномозкової пункції стан різко погіршився: виникли судоми, втрата свідомості, аритмічне дихання, анізокорія. Яка найбільш імовірна причина погіршення стану? The child's condition worsened 4 hours after the spinal tap: convulsions, loss of consciousness, arrhythmic breathing, anisocoria occurred. What is the most likely cause of the deterioration?

Гострий розлад мозкового кровообігу Acute cerebral circulation disorder

Внутрішньочерепний крововилив Intracranial hemorrhage

Епілептичний напад Epileptic attack

Дислокація та уклинення мозку Dislocation and entrapment of the brain

Істеричний напад Hysterical attack

56 / 200
Хвора 35-ти років перебуваючи у переповненому тролейбусі у середині натовпу, відчула запаморочення, слабкість, після чого знепритомніла. На зупинці прийшла до тями. Об’єктивно: пацієнтка бліда, шкіра на лобі волога, Ps- 90/хв., АТ- 90/65 мм рт.ст. Розміри серця в нормі, тони серця дещо приглушені. В анамнезі хронічних захворювань немає. Який діагноз найбільш імовірний? A 35-year-old patient, being in a crowded trolleybus in the middle of a crowd, felt dizzy, weak, and then fainted. She regained consciousness at the stop. Objectively: the patient pale, the skin on the forehead is moist, Ps- 90/min., BP- 90/65 mm Hg. Heart sizes are normal, heart sounds are somewhat muffled. There is no history of chronic diseases. What is the most likely diagnosis?

Анемія, анемічний колапс Anemia, anemic collapse

Вегето-судинна дистонія за гіпотонічним типом, зомління Vegeto-vascular dystonia of the hypotonic type, grinding

Ішемічна хвороба серця, інфаркт міокарда Ischemic heart disease, myocardial infarction

Гострий мозковий інсульт Acute stroke

Епілепсія (малий напад) Epilepsy (small attack)

57 / 200
До приймального відділення лікарні доставлено хлопчика, якого півгодини тому вкусила гадюка. Потерпілий скаржиться на біль у правій гомілці, сонливість, кволість. У нижній третині гомілки є дві невеличкі ранки з вираженим набряком навколо та синцями навкруги ранок. Які маніпуляції НЕ МОЖНА виконувати потерпілому при наданні невідкладної допомоги? A boy who was bitten by a viper half an hour ago was brought to the hospital's reception department. The victim complains of pain in the right leg, drowsiness, weakness. There are two small wounds in the lower third of the leg with pronounced swelling around and bruises around the wound. What manipulations CANNOT be performed on the victim during emergency care?

Виконувати циркулярну новокаїнову блокаду кінцівки Perform circular novocaine blockade of the limb

Надавати спокій кінцівці Give rest to the limb

Накладати артеріальний джгут Apply an arterial tourniquet

Вводити полівалентну протизміїну сироватку Inject polyvalent antiserum

Накладати венозний джгут Apply venous tourniquet

58 / 200
Хворому 15-ти років проведена амбулаторно 16 годин тому закрита репозиція свіжого перелому обох кісток лівого передпліччя в нижній третині, накладена глибока гіпсова лонгета від середньої третини плеча. Скарги на біль та анестезію в кисті. Об’єктивно: пальці холодні, малорухомі, бліді. Яке ускладнення виникло у хворого? A 15-year-old patient underwent a closed reduction of a fresh fracture of both bones of the left forearm in the lower third on an outpatient basis 16 hours ago, a deep plaster splint was applied from the middle third of the shoulder. Complaints about pain and anesthesia in the hand. Objectively: the fingers are cold, immobile, pale. What complications did the patient experience?

Травматичний неврит ліктьового нерва Traumatic neuritis of the ulnar nerve

Гострий тромбоз плечової артерії Acute brachial artery thrombosis

Гострий травматичний тромбофлебіт вен плеча Acute traumatic thrombophlebitis of shoulder veins

Лімфостаз Lymphostasis

Ішемічна контрактура Фолькмана Volkman ischemic contracture

59 / 200
Хворий 24-х років звернувся у поліклініку до хірурга з різаною раною кисті, яку отримав дома 1 годину тому. Хірургом виконана первинна хірургічна обробка рани. Із слів хворого 3 роки тому був повністю щеплений від правця, але документи про це відсутні. Яка тактика лікаря? A 24-year-old patient went to the polyclinic with a cut wound on the hand, which he received at home 1 hour ago. The surgeon performed primary surgical treatment of the wound. According to the patient 3 I was fully vaccinated against tetanus years ago, but there are no documents about it. What are the doctor's tactics?

Ввести 250 ОД протиправцевого імуноглобуліну людини Enter 250 units of human anti-tetanus immunoglobulin

Ввести 0,5 мл правцевого анатоксину 250 ОД протиправцевого імуноглобуліну людини Inject 0.5 ml of tetanus toxoid with 250 units of human anti-tetanus immunoglobulin

Обколоти рану антибіотиком Apply the wound with an antibiotic

Ввести 1,0 мл правцевого анатоксину Enter 1.0 ml of tetanus toxoid

Нічого не робити Do nothing

60 / 200
Дівчинка 4-х років скаржиться на загальну слабкість, кашель. Вночі раптово підвищилася температура до 39,8oC, хвора стала неспокійною. Гавкаючий кашель, шумне свистяче дихання з втягуванням над- та підключичних западин, міжреберних проміжків, шкіра бліда. Контактувала з хворими на ГРВІ сестрами. Що необхідно призначити дитині в першу чергу? A 4-year-old girl complains of general weakness, cough. At night, the temperature suddenly rose to 39.8oC, the patient became restless. Barking cough, noisy wheezing with inhalation supraclavicular and subclavian depressions, intercostal spaces, the skin is pale. I contacted sisters with SARS. What should be prescribed to the child first of all?

Папаверин, вітамін С Papaverine, vitamin C

Антибіотики, димедрол Antibiotics, diphenhydramine

Преднізолон, гарячі ніжні ванни Prednisone, hot gentle baths

Дибазол, еуфілін Dibazol, Euphilin

Астмопент, діазолін Astmopent, diazolin

61 / 200
Хвора 27-ми років надійшла в ургентному порядку в лікарню зі скаргами на утруднене дихання, нудоту, блювання, свербіж шкіри, що з’явилися годину тому після ін’єкції пеніциліну. Об’єктивно: загальний стан важкий, на шкірі уртикарний висип. Дихання стридорозне, тони серця ослаблені. Ps- і20/хв., АТ- 80/50 мм рт.ст. Який із запропонованих заходів слід провести в першу чергу? A 27-year-old patient came urgently to the hospital with complaints of difficulty breathing, nausea, vomiting, itching of the skin, which appeared an hour ago after other' injections of penicillin. Objectively: the general condition is severe, urticarial rash on the skin. Breathing is stridorous, heart sounds are weakened. Ps - 20/min., BP - 80/50 mm Hg. Which of the proposed measures should be carried out first?

Ін’єкція димедролу Diphenhydramine injection

Киснева терапія Oxygen therapy

Введення еуфіліну Introducing Euphilin

Введення натрію броміду Introduction of sodium bromide

Конікотомія Conicotomy

62 / 200
У вагітної 29-ти років у приймальному відділенні виникли біль у епіга-стральній ділянці, нудота, блювання, мерехтіння 'мушок'перед очима. АТ-170/100 мм рт.ст. на обох руках; визначаються генералізовані набряки. Який найбільш вірогідний діагноз? A 29-year-old pregnant woman in the reception department developed pain in the epigastric area, nausea, vomiting, flickering of 'flies' before her eyes. AT-170/100 mm Hg on both hands; generalized edema is detected. What is the most likely diagnosis?

Прееклампсія важкого ступеня Severe preeclampsia

Еклампсія Eclampsia

Загроза крововиливу в мозок Threat of cerebral hemorrhage

Відшарування сітківки ока Retinal detachment

Прееклампсія середнього ступеня Moderate preeclampsia

63 / 200
У породіллі в ранній післяпологовий період виникла гіпотонічна кровотеча, яка склала 15% від маси тіла, кровотеча продовжується. Об’єктивно: шкірні покрови бліді, акроціаноз, жінка адинамічна. Ps- 130/хв., АТ- 75/50 мм рт.ст. Порушень з боку системи зсідання крові немає. Олігу-рія. Яка тактика лікаря? In the early postpartum period, hypotonic bleeding occurred in the early postpartum period, which amounted to 15% of the body weight, the bleeding continues. Objectively: the skin is pale, acrocyanosis, the woman is adynamic . Ps- 130/min., BP- 75/50 mm Hg. There are no disorders on the part of the blood coagulation system. Oliguria. What are the doctor's tactics?

Перев’язка маткових судин Ligation of uterine vessels

Екстирпація матки Uterus extirpation

Надпіхвова ампутація матки Supravaginal amputation of the uterus

Накласти затискачі на шийку матки Put clamps on the cervix

Перев’язка внутрішньої клубової артерії Internal iliac artery ligation

64 / 200
До реанімаційного відділення бригадою швидкої допомоги доставлено хлопчика 15-ти років, який під час катання на ковзанах потрапив у ополонку. Об’єктивно: ціаноз, з дихальних шляхів виділяється рожеве харкотиння. АТ- 160/110 мм рт.ст., тахіаритмія, в легенях - різнокаліберні вологі хрипи. Який найбільш імовірний діагноз? A 15-year-old boy was brought to the intensive care unit by the ambulance team, who fell into a sinkhole while skating. Objectively: cyanosis, respiratory tract discharge pink sputum. Blood pressure - 160/110 mm Hg, tachyarrhythmia, wet rales of various calibers in the lungs. What is the most likely diagnosis?

Асфіктичне утоплення у прісній воді Asphyxic drowning in fresh water

Істинне утоплення в морській воді True drowning in sea water

Синкопальне утоплення у прісній воді Syncopal drowning in fresh water

Асфіктичне утоплення у морській воді Asphyxic drowning in sea water

Істинне утоплення у прісній воді True freshwater drowning

65 / 200
У хворого через 5 днів після екстракції зуба виникли лихоманка, задишка, біль у ділянці серця. Об’єктивно: шкіра кольору 'кави з молоком', to- 39,8oC, Ps- 100/хв., ритм 'перепілки', систолічний і мезодіастолічний шуми на верхівці, акцент ІІ тону над легеневим стовбуром. Печінка +4 см, селезінка +2 см. На шкірі гомілок - геморагічна висипка. Який з методів є найбільш доцільним для встановлення діагнозу? Five days after tooth extraction, the patient developed fever, shortness of breath, pain in the heart area. Objectively: skin the color of 'coffee with milk', to- 39 ,8oC, Ps- 100/min., 'quail' rhythm, systolic and mesodiastolic murmurs at the apex, emphasis of the II tone over the pulmonary trunk. Liver +4 cm, spleen +2 cm. On the skin of the lower legs - a hemorrhagic rash. Which of the methods is the most appropriate for establishing a diagnosis?

Електрокардіографія Electrocardiography

Посів крові на гемокультуру Blood culture for hemoculture

Прискорена реакція на сифіліс Accelerated response to syphilis

УЗД печінки та селезінки Ultrasound of liver and spleen

Загальний аналіз крові General blood test

66 / 200
Хворий 50-ти років після аварії на виробництві був госпіталізований із скаргами на металевий присмак та печіння у роті, нудоту, слинотечу, багаторазове блювання, пронос з домішками слизу та крові, біль у епіга-стральній та поперековій ділянках. Об’єктивно: стан важкий, виразковий стоматит, гінгівіт, некроз слизової оболонки носа, набряк гортані. АТ-150/100 мм рт.ст., Ps- 48/хв, тони серця глухі. Поліурія, гіпостенурія, альбумінурія, гематурія. Який антидот слід ввести хворому? A 50-year-old patient was hospitalized after an industrial accident with complaints of a metallic taste and burning in the mouth, nausea, drooling, repeated vomiting, diarrhea with mucus impurities and of blood, pain in the epigastric and lumbar areas. Objectively: the condition is severe, ulcerative stomatitis, gingivitis, necrosis of the nasal mucosa, swelling of the larynx. BP-150/100 mm Hg, Ps- 48/min, heart sounds deaf. Polyuria, hyposthenuria, albuminuria, hematuria. What antidote should be administered to the patient?

Атропіну сульфат Atropine sulfate

Антарсин Antarsyn

Дієтиоксим Dietioxime

Унітіол Unithiol

Хромосмон Chromosmon

67 / 200
У вагітної в терміні 37 тижнів раптово погіршився стан. В анамнезі гіпертонічна хвороба, цукровий діабет. Скаржиться на різкий біль у животі та кров’янисті виділення з піхви. Об’єктивно: шкіра та слизові бліді, пульс ниткоподібний, до 110/хв., АТ-90/50 мм рт.ст., матка в підвищеному тонусі, болісна під час пальпації в ділянці розташування плаценти. Серцебиття плоду відсутнє, з піхви продовжується виділення темної крові. Який найбільш імовірний діагноз? A 37-week pregnant woman's condition suddenly worsened. She has a history of hypertension and diabetes. She complains of sharp abdominal pain and vaginal bleeding. About Objectively: the skin and mucous membranes are pale, the pulse is threadlike, up to 110/min, blood pressure 90/50 mmHg, the uterus is in increased tone, painful during palpation in the area of ​​the placenta. There is no fetal heartbeat, discharge from the vagina continues dark blood. What is the most likely diagnosis?

Розрив матки, що розпочався Initiated uterine rupture

Передлежання плаценти Placenta previa

Розрив крайового синусу плаценти Rupture of marginal sinus of placenta

Загроза розриву матки Threat of uterine rupture

Передчасне відшарування плаценти Premature placental abruption

68 / 200
У 2-річної дитини невдовзі після щеплення з’явились петехіальний висип на шкірі, носова кровотеча, крововиливи у періорбітальні ділянки. Інших патологічних змін не виявлено. У крові: лейкоцити - 5 • 109/л, Hb-110 г/л, тромбоцити - 3 • 109/л. Що найімовірніше стало причиною геморагічного синдрому в дитини? Shortly after vaccination, a 2-year-old child developed a petechial rash on the skin, nosebleeds, hemorrhages in the periorbital areas. No other pathological changes were detected. In the blood: leukocytes - 5 • 109/l, Hb-110 g/l, platelets - 3 • 109/l. What was the most likely cause of hemorrhagic syndrome in a child?

Гострий лейкоз Acute leukemia

Дисеміноване внутрішньосудинне зсідання крові Disseminated intravascular coagulation

Геморагічний васкуліт Hemorrhagic vasculitis

Імунна тромбоцитопенічна пурпура Immune thrombocytopenic purpura

Системний червоний вівчак System red sheep

69 / 200
13- річна хвора скаржиться на сильний пульсуючий біль у лівій половині голови, якому передує поява райдужних плям в лівих полях зору. Напад триває декілька годин, супроводжується нудотою. На подібні напади страждає мати хворої. АТ-100/70 мм рт.ст, Ps- 60/хв. Який найбільш імовірний діагноз? A 13-year-old patient complains of severe throbbing pain in the left half of the head, which is preceded by the appearance of iridescent spots in the left field of vision. The attack lasts several hours, accompanied by nausea. the patient's mother suffers from similar attacks. Blood pressure - 100/70 mm Hg, Ps - 60/min. What is the most likely diagnosis?

Мігренозний приступ Migraine attack

Симпато-адреналовий криз Sympatho-adrenal crisis

Ретробульбарний неврит Retrobulbar neuritis

Транзиторна ішемічна атака Transient ischemic attack

Невралгія трійчастого нерва Trigeminal neuralgia

70 / 200
Хвора 32-х років перенесла крупозну пневмонію. На даний час скаржиться на озноб, задишку в спокої, загальну слабкість, головний біль, підвищення to до 39,5°С', пітливість. Об’єктивно: зміщення назовні лівої межі серця, тони серця значно ослаблені, систолічний шум на верхівці, АТ- 80/50 мм рт.ст. У крові: лейк.- 10,5 • 109/л, е- 8%, ШЗЕ- 45 мм/год. У сечі: білок - 0,066 г/л, ер.- 15-20 у п/з, гіалінові циліндри - 4-5 в п/з. Яке ускладнення виникло у хворої? A 32-year-old patient suffered from croup pneumonia. Currently, she complains of chills, shortness of breath at rest, general weakness, headache, temperature rise to 39.5° C', sweating. Objectively: outward displacement of the left border of the heart, heart sounds are significantly weakened, systolic murmur at the apex, blood pressure - 80/50 mm Hg. In the blood: leuk. - 10.5 • 109/l, e - 8%, SZE - 45 mm/h. In urine: protein - 0.066 g/l, er. - 15-20 in p/z, hyaline cylinders - 4-5 in p/z. What complication occurred in the patient?

Інфекційно-токсичний шок Infectious-toxic shock

Інфаркт міокарда Myocardial infarction

Тромбоемболія легеневої артерії Thromboembolism of the pulmonary artery

Кардіоміопатія Cardiomyopathy

Перикардит Pericarditis

71 / 200
Хворий 28-ми років, ін’єкційний наркоман, скаржиться на переміжну гарячку, пітливість, слабкість, тривалий пронос, герпетичне враження порожнини рота, кашель. Втрата маси тіла перевищує 10%. Об’єктивно: блідий, зниженого харчування. Збільшені периферичні лімфовузли, рухомі, неболючі. Слизова оболонка порожнини рота вкрита білим нальотом, герпетичний висип. Збільшені печінка та селезінка. В легенях вислуховуються сухі та вологі хрипи. Який найбільш імовірний діагноз? A 28-year-old patient, an injection drug addict, complains of intermittent fever, sweating, weakness, prolonged diarrhea, herpetic impression of the oral cavity, cough. Weight loss exceeds 10%. Objectively: pale, undernourished. Peripheral lymph nodes are enlarged, mobile, painless. Mucous membrane of the oral cavity is covered with white plaque, herpetic rash. Liver and spleen are enlarged. Dry and moist rales are heard in the lungs. What is the most likely diagnosis?

Дизентерія Dysentery

Пре-СНІД Pre-AIDS

Пневмонія Pneumonia

Лімфогранулематоз Lymphogranulomatosis

Дифтерія Diphtheria

72 / 200
Хворий 52-х років сонливий, загальмований. З рота 'печінко-вий'запах, тричі було блювання. Хворіє на виразкову хворобу шлунка. 2 місяці тому по поводу шлункової кровотечі лікувався, були гемотрансфу-зії. Стан важкий, інтенсивна жовтяниця, на слизових оболонках геморагії. Печінка визначається вище реберної дуги. Ps- 110/хв., АТ90 /60 мм рт.ст. Вкажіть наймовірніший невідкладний стан : The 52-year-old patient is drowsy, inhibited. He has a 'livery' smell from his mouth, he vomited three times. He suffers from gastric ulcer disease. 2 months ago due to gastric bleeding was treated, hemotransfusions were performed. The condition is severe, intense jaundice, hemorrhages on the mucous membranes. The liver is defined above the costal arch. Ps- 110/min., BP90 /60 mmHg. Specify the most likely emergency condition :

Гостра ниркова недостатність Acute renal failure

Гостра дихальна недостатність Acute respiratory failure

Інфекційно-токсичний шок Infectious-toxic shock

Гостра печінкова енцефалопатія Acute hepatic encephalopathy

Дегідратаційний шок Dehydration shock

73 / 200
У хворого 47-ми років останні дві доби відсутні сечовипускання та позиви до нього. 8 років тому назад переніс лівосторонню нефректомію у зв’язку з ушкодженням нирки. Останній місяць відмічав тупий біль у правій поперековій ділянці, який набув характеру ниркової кольки три доби тому назад. Після застосування спазмо-аналгетиків біль майже зник. Який результат лабораторного обстеження має вирішальне значення для вибору лікування хворого? A 47-year-old patient has had no urination and no urge to urinate for the past two days. 8 years ago, he underwent a left-sided nephrectomy due to kidney damage. The last month noted a dull pain in the right lumbar area, which took on the character of renal colic three days ago. After the use of antispasmodic analgesics, the pain almost disappeared. What result of the laboratory examination is of decisive importance for the selection of the patient's treatment?

Гемоглобін крові Blood hemoglobin

Кальцій крові Blood calcium

Сечовина, креатинін крові Urea, blood creatinine

Калій крові Blood potassium

Натрій крові Blood sodium

74 / 200
У хворої 42-х років гострий лівобічний пієлонефрит ускладнився бактеріальним шоком та гострою нирковою недостатністю. Дві доби тому настала поліурична стадія. Які зміни в організмі слід попередити в першу чергу? In a 42-year-old patient, acute left-sided pyelonephritis was complicated by bacterial shock and acute renal failure. Two days ago, the polyuric stage began. What changes in the body should be prevented first of all ?

Гіперазотемія Hyperazotemia

Гіпокаліємія Hypokalemia

Гіпоальбумінемія Hypoalbuminemia

Анемія Anemia

Набряковий синдром Oedema syndrome

75 / 200
У породіллі 34-х років, на 6-ту добу після пологів, раптово з’явився тупий біль розпираючого характеру в крижово-поперековій ділянці, нижніх відділах живота та правому стегні. Поступово став наростати набряк правої ноги у ділянці стегна та гомілки, ціаноз шкіри. Температура підвищилася до 38,5oC. Яке ускладнення виникло в хворої? A 34-year-old woman in labor, on the 6th day after giving birth, suddenly experienced a dull, distending pain in the sacro-lumbar area, lower abdomen and right thigh. The swelling of the right leg in the area of ​​the thigh and lower leg gradually increased, cyanosis of the skin. The temperature rose to 38.5oC. What complication did the patient experience?

Гострий тромбофлебіт тазових вен Acute thrombophlebitis of pelvic veins

Гострий післяпологовий параметрит Acute postpartum parametritis

Синя флегмазія Blue phlegmasia

Біла флегмазія White phlegmasia

Гострий ілеофеморальний флебо-тромбоз Acute iliofemoral phlebo-thrombosis

76 / 200
З вогнища хімічного ураження, утвореного зарином, до МПП доставлено ураженого. Відзначаються напади ядухи за типом бронхіальної астми, фібрилярні посмикування жувальних м’язів. Який антидот необхідно ввести ураженому разом з атропіном на даному етапі медичної евакуації? From the focus of chemical damage caused by sarin, the affected person was brought to the MPP. There are attacks of dyspnoea according to the type of bronchial asthma, fibrillary twitching of the masticatory muscles. What antidote should be administered to the affected person along with atropine at this stage of medical evacuation?

Тіосульфат натрію Sodium thiosulfate

Антиціан Antician

Дипіроксим Dipiroxime

Амілнітрит Amyl nitrite

Унітіол Unithiol

77 / 200
Хворий 42-х років з приводу ревматоїдного артриту отримував преднізолон впродовж 4-х місяців. На фоні диспептичних розладів відмінив прийом преднізолону самостійно. Через 2 дні з’явилися виражена загальна слабкість, болі у суглобах і животі. Об’є- ктивно : млявий, адинамічний, шкірні покриви бліді з мармуровим відтінком. Пальпація кишечнику болісна. ЧСС- 72/хв., АТ- 60/20 мм рт.ст. Натрій сироватки крові - 90 ммоль/л, калій - 6,6 ммоль/л, рівень глюкози крові -4,6 ммоль/л, рівень глюкози крові - 4,6 ммоль/л. Чим зумовлений стан хворого? A 42-year-old patient received prednisone for 4 months due to rheumatoid arthritis. Against the background of dyspeptic disorders, he stopped taking prednisone on his own. After 2 days, severe general weakness, pains in the joints and abdomen. Objectively: lethargic, adynamic, the skin is pale with a marble shade. Palpation of the intestines is painful. Heart rate - 72/min., BP - 60/20 mm Hg. Sodium in blood serum - 90 mmol/l, potassium - 6.6 mmol/l, blood glucose level -4.6 mmol/l, blood glucose level - 4.6 mmol/l. What is the cause of the patient's condition?

Гостра тиреоїдна недостатність Acute thyroid insufficiency

Гіпоглікемічний криз Hypoglycemic crisis

Шлунково-кишкова кровотеча Gastrointestinal bleeding

Синдром відміни глюкокортикоїдів Glucocorticoid withdrawal syndrome

Судинний колапс Vascular collapse

78 / 200
Дитина 5-ти місяців доставлена в реанімаційне відділення у зв’язку з вираженими явищами ексикозу, токсикозу і затримкою сечі протягом 20-ти годин. За останні 2 доби відзначалися 10-разові рідкі випорожнення, а за останню добу - 3-разове блювання. Адекватною стартовою терапією слід вважати: A 5-month-old child was taken to the intensive care unit due to pronounced symptoms of exicosis, toxicosis and retention of urine for 20 hours. Over the past 2 days, there have been 10 times liquid stools, and in the last day - 3 times vomiting. Adequate starting therapy should be considered:

Ізотонічний розчин NаCl Isotonic NaCl solution

Трансфузія глюкози Glucose transfusion

Спазмолітики Spasmolytics

Трансфузія препаратів крові Transfusion of blood products

Сечогінні препарати Diuretics

79 / 200
У пацієнтки 55-ти років 5 років тому діагностовано цироз печінки. Лікувалась нерегулярно. Доставлена з клінікою шлунково-кишкової кровотечі. Під час фіброезофагогастродуоденоскопії виявлено розширені вени нижньої третини стравоходу з кровотечею з них. Який з методів лікування слід застосувати в першу чергу? A 55-year-old patient was diagnosed with liver cirrhosis 5 years ago. She was treated irregularly. She was brought to the clinic for gastrointestinal bleeding. During fibroesophagogastroduodenoscopy, dilated veins of the lower third of the esophagus were detected with bleeding from them. Which of the treatment methods should be applied first?

Оперативне втручання у невідкладному порядку Urgent intervention

Перевести хвору до реанімаційного відділення Transfer the patient to the intensive care unit

Госпіталізувати в хірургічне відділення і призначити сечогінні і гепатопротектори Hospitalize in the surgical department and prescribe diuretics and hepatoprotectors

Госпіталізувати в хірургічне відділення і призначити гемостатичну і кровозамісну терапію Hospitalize in the surgical department and prescribe hemostatic and blood replacement therapy

Госпіталізувати в хірургічне відділення, поставити зонд Блекмора, призначити гемостатичну і кровозамі-сну терапію Hospitalize in the surgical department, insert a Blackmore probe, prescribe hemostatic and blood replacement therapy

80 / 200
Чоловік 78-ми років викликав швидку допомогу з приводу блюван- ня, здуття живота, наявності болючого грижового випинання у паху, яке не вправляється у черевну порожнину зі вчорашнього дня. Під час огляду лікарем швидкої допомоги грижове випинання вправилося, 6іль пройшов. Які дії лікаря? A 78-year-old man called an ambulance due to vomiting, abdominal distension, the presence of a painful hernial protrusion in the groin, which has not been inserted into the abdominal cavity since yesterday . During the examination by the emergency doctor, the hernial protrusion became visible, 6il passed. What actions did the doctor take?

Протягом години спостерігати за динамікою клінічної картини Over the course of an hour, observe the dynamics of the clinical picture

Рекомендувати хірургічне лікування у плановому порядку Recommend surgical treatment as planned

Призначити амбулаторне обстеження Schedule outpatient examination

Доставити хворого до стаціонару Deliver the patient to the hospital

Призначити дієту і холод на живіт Prescribe a diet and a cold on the stomach

81 / 200
Потерпілий був збитий автомашиною. У верхній третині лівого стегна рана 4х10 см, з якої виступає уламок кістки, значна варусна деформація стегна. Виберіть оптимальний метод іммобілізації: The victim was hit by a car. There is a 4x10 cm wound in the upper third of the left thigh, from which a bone fragment protrudes, significant varus deformation of the thigh. Choose the optimal method of immobilization:

Шини Дітеріхса та Крамера Tires of Dieterichs and Kramer

Дві драбинчасті шини Two ladder tires

Шина Томаса Thomas Tire

Три драбинчасті шини Three ladder tires

Шина Дітеріхса Tire of Dieterichs

82 / 200
Пацієнт 42-х років з тривалим 'ви-разковим'анамнезом доставлений у важкому стані із зниженням артеріального тиску до 90/60 мм рт.ст., судомами, явищами гастральної тетанії. На контрастній рентгенографії шлунка діагностовано субкомпенсований пілородуоденальний стеноз. Яку лікувальну тактику слід обрати? A 42-year-old patient with a long-term 'ulcer' anamnesis was delivered in serious condition with a drop in blood pressure to 90/60 mmHg, convulsions, phenomena of gastric tetany. Subcompensated pyloroduodenal stenosis was diagnosed on contrast X-ray of the stomach. What treatment tactics should be chosen?

Ургентне бужування пілоросте-нозу за допомогою фіброезофагогастродуоденоскопа Urgent bulging of the serous nose using a fibroesophagogastroduodenoscope

Короткотривала інтенсивна терапія з наступною резекцією шлунка за одним із методів Short-term intensive therapy followed by gastric resection using one of the methods

Невідкладне хірургічне втручання -резекція шлунка за одним з методів Urgent surgical intervention - resection of the stomach by one of the methods

Комплексна противиразкова терапія Complex anti-ulcer therapy

Тривала інтенсивна інфузійна терапія для корекції водно-електролітного, білкового, вуглеводного та жирового балансів Long-term intensive infusion therapy to correct water-electrolyte, protein, carbohydrate and fat balances

83 / 200
Лікар швидкої медичної допомоги, оглянувши дитину віком 5 місяців через дві години після травми, виставив діагноз: термічний опік іі-ііі ступеня нижніх кінцівок, промежини, спини до 20%, опіковий шок іі ступеня. Яким буде об’єм медичної допомоги? The emergency medical doctor, after examining a 5-month-old child two hours after the injury, made a diagnosis: thermal burn of the ii-iii degree of the lower extremities, perineum, back up to 20 %, second-degree burn shock. What will be the amount of medical assistance?

Накладання асептичної пов’язки Applying an aseptic bandage

Новокаїнові блокади, асептична пов’язка на уражені ділянки Novocaine blocks, aseptic bandage on affected areas

Інфузійна терапія Infusion therapy

Знеболювання, інфузійна терапія, накладання асептичної пов’язки Analgesia, infusion therapy, aseptic dressing

Охолодження опікової поверхні Cooling of the burn surface

84 / 200
Жінка 26-ти років з терміном вагітності 6-7 тижнів скаржиться на нудоту та блювання до 3- х разів на добу. За останні 3 дні блювання почастішало до 15-18 разів на добу, жінка схудла на 3,5 кг, з’явилась різка слабкість. Об’єктивно: шкіра бліда, тургор тканин знижений, Ps- 100/хв., АТ- 100/50 мм рт.ст., температура тіла 36,8oC. У сечі: реакція на ацетон позитивна (++). Який найбільш імовірний діагноз? A 26-year-old woman with a pregnancy period of 6-7 weeks complains of nausea and vomiting up to 3 times a day. Over the past 3 days, vomiting has increased to 15 - 18 times a day, the woman lost weight by 3.5 kg, sharp weakness appeared. Objectively: pale skin, decreased tissue turgor, Ps- 100/min., BP- 100/50 mm Hg, temperature body 36.8oC. In urine: the reaction to acetone is positive (++). What is the most likely diagnosis?

Блювання вагітних Pregnant vomiting

Гострий панкреатит Acute pancreatitis

Гострий холецистит Acute cholecystitis

Гострий гастрит Acute gastritis

Харчова токсикоінфекція Food poisoning

85 / 200
У хворого 22-х років, що страждає на шизофренію, без зовнішньої причини виник стан з руховим збудженням, нецілеспрямованими стереотипними рухами, раптовими, несподіваними імпульсивними діями з агресією, люттю, руйнівними діями, кривлянням, гримасуванням, негативізмом, незв’язною мовою. Які препарати з наведених, насамперед, доцільно використати для усунення цього невідкладного стану? A 22-year-old patient suffering from schizophrenia, without an external cause, developed a condition with motor excitement, non-targeted stereotyped movements, sudden, unexpected impulsive actions with aggression, rage, destructive actions, grimacing, grimacing, negativism, incoherent language. Which of the following drugs should be used primarily to eliminate this emergency?

Транквілізатори Tranquilizers

Нейролептики Neuroleptics

Судинні препарати Vascular preparations

Ноотропи Nootropics

Антидепресанти Antidepressants

86 / 200
У дитини 2-х років дифтерійний круп. На 2-й день серотерапії виникла зупинка дихання. Що стало причиною асфіксії? A 2-year-old child has diphtheria croup. On the 2nd day of serotherapy, breathing stopped. What caused the asphyxia?

Стеноз гортані Laryngeal stenosis

Парез дихальної мускулатури Paresis of respiratory muscles

Сироваткова хвороба Serum sickness

Механічна обтурація плівками Mechanical obturation with films

Анафілактичний шок Anaphylactic shock

87 / 200
У роділлі 25-ти років після відхо- дження навколоплідних вод серцебиття плоду сповільнилось до 100/хвили-ну та після перейм не вирівнюється. При вагінальному дослідження виявлено: шийка матки згладжена, розкриття маткового вічка повне, плідного міхура немає. Передлежить голівка в порожнині малого тазу. Стрілоподібний шов у правому косому розмірі, мале тім’ячко зліва. Яка тактика лікаря в даній ситуації? In a 25-year-old woman in labor, after the discharge of amniotic fluid, the fetal heartbeat slowed down to 100/minute and did not level off after delivery. A vaginal examination revealed: the cervix the uterus is smoothed, the opening of the uterine cavity is complete, there is no amniotic sac. The head is in the pelvic cavity. The arrow-shaped suture is in the right oblique dimension, the small cap is on the left. What are the doctor's tactics in this situation?

Кесарів розтин Caesarean section

Підсилення пологової діяльності Intensification of reproductive activity

Лікування гіпоксії плода Treatment of fetal hypoxia

Накладання бинта Вербова Applying Verbov's bandage

Накладання порожнинних акушерських щипців Applying cavity obstetric forceps

88 / 200
Бригада швидкої допомоги викликана до повторнонароджуючої жінки, що народила вдома. Матка на рівні пупка, кульоподібної форми, з піхви звисає пуповина. Після народження дитини пройшло 30 хвилин. Яка тактика лікаря? An ambulance was called to a woman who gave birth again at home. The uterus is at the level of the navel, spherical in shape, the umbilical cord hangs from the vagina. 30 minutes have passed since the birth of the child. What doctor's tactics?

Ввести в/в окситоцин Inject intravenous oxytocin

Виконати ручне відділення посліду Perform manual litter separation

Негайно транспортувати жінку до пологового відділення Immediately transport the woman to the maternity ward

Застосувати метод Абуладзе Apply the Abuladze method

Продовжити очікування до 2-х годин Continue waiting up to 2 hours

89 / 200
У хворого 32-х років раптово виникли озноб, блювання, судоми, підвищення температури до 40oC. За кілька годин з’явилися ригідність м’язів потилиці та симптоми Керніга, оглушення (9 балів за шкалою Глазго), анурія. Виявлені ціаноз шкірних покривів, поодинокі геморагічні висипання на шкірі, АТ- 70/40 мм рт.ст., пульс ниткоподібний, 110 /хв. Яке ускладнення виникло у хворого? A 32-year-old patient suddenly developed chills, vomiting, convulsions, an increase in temperature to 40oC. In a few hours, stiffness of the muscles of the back of the head and Kernig's symptoms appeared , stupor (9 points according to the Glasgow scale), anuria. Cyanosis of the skin, isolated hemorrhagic rashes on the skin, blood pressure 70/40 mm Hg, thread-like pulse, 110/min. What complications occurred in the patient?

Ішемічний інсульт Ischemic stroke

Набряк і вклинення мозку в потиличний отвір Swelling and wedging of the brain in the occipital foramen

Бульбарний синдром Bulbar syndrome

Тромбоз кавернозного синусу Thrombosis of the cavernous sinus

Синдром Уотерхауса-Фридериксена Waterhouse-Friederiksen syndrome

90 / 200
Бригада швидкої допомоги прибула на місце ДТП. Постраждалий знаходиться в салоні автомобіля, притомний, на біль не скаржиться, не може поворухнути ні руками, ні ногами, на лівій гомілці по передній поверхні ра- на, що помірно кровоточить. З якої дії необхідно почати надання медичної допомоги? The ambulance team arrived at the scene of the accident. The victim is in the passenger compartment of the car, he is conscious, he does not complain of pain, he cannot move his arms or legs, on his left leg on the front surface of a moderately bleeding wound. What action should be taken to provide medical assistance?

Проведення допоміжної вентиляції легень Carrying out auxiliary lung ventilation

Накладення кровоспинного джгута Applying a tourniquet

Накладення шийного комірця Applying neck collar

Інгаляція кисню Oxygen inhalation

Вилучити постраждалого з салону автомобіля Remove the victim from the car

91 / 200
У дитини, що потрапила в автомобільну аварію, є ознаки порушеної перфузії: систолічний тиск 60 мм рт.ст. Оберіть оптимальне лікування: A child involved in a car accident has signs of impaired perfusion: systolic pressure 60 mm Hg. Choose the optimal treatment:

Інфузія білкових препаратів Infusion of protein preparations

Інфузія сольових розчинів Saline infusion

Інфузія допаміну Dopamine infusion

Призначення серцевих глікозидів Prescription of cardiac glycosides

Призначення атропіну Prescription of atropine

92 / 200
Чоловік 36-ти років раптово спалив на собі волосся. Потім схопив свою 12-річну доньку і хотів скинути її з балкону. Був затриманий родичами. Свій вчинок пояснив тим, що 'виконував наказ, який звучав у нього в голові'. Визначте тактику лікаря-терапевта, якого викликали родичі: A 36-year-old man suddenly burned his hair. Then he grabbed his 12-year-old daughter and wanted to throw her off the balcony. He was detained by relatives. He explained his act by , who 'carried out the order that sounded in his head.' Define the tactics of the therapist who was called by the relatives:

Призначити хворому лікування вдома Prescribe the patient treatment at home

Зафіксувати хворого до моменту заспокоєння Fix the patient until the moment of sedation

Викликати спеціалізовану бригаду швидкої медичної допомоги Call a specialized emergency medical team

Терміново госпіталізувати в психіатричну лікарню, використавши транспорт родичів Urgent hospitalization in a psychiatric hospital, using relatives' transport

Оформити виклик консультанта лікаря-психіатра Call a consultant psychiatrist

93 / 200
У дитини 11-ти років на п’ятий день від початку збільшення привушних слинних залоз температура підвищилася до 39oC, виникли головний біль, блювання до 6 разів на добу. Запідозрений паротитний менінгіт. Який з перелічених варіантів найбільш інформативний для встановлення остаточного діагнозу: паротитний менінгіт? In an 11-year-old child, on the fifth day from the beginning of the increase in the parotid salivary glands, the temperature rose to 39oC, there was a headache, vomiting up to 6 times a day. Suspected parotid meningitis. Which of the listed options is the most informative for establishing the final diagnosis: parotid meningitis?

Виділення вірусу паротиту з ліквору Isolation of mumps virus from cerebrospinal fluid

Люмбальна пункція та наявність лімфоцитарного цитозу Lumbar puncture and the presence of lymphocytic cytosis

Позитивні менінгеальні симптоми Positive meningeal symptoms

Відсутність токсикозу при виражених ознаках внутрішньочерепної гіпертензії Absence of toxicosis with pronounced signs of intracranial hypertension

Чотириразове зростання титру специфічних антитіл у реакції зв’язування комплементу Fourfold increase in the titer of specific antibodies in the complement binding reaction

94 / 200
У хворого на епілепсію з’явились постійні великі судомні напади з тонічними і клонічними судомами, між якими хворий залишається непритомним. На доторкання і больові подразники не реагує. Зіниці вузькі, реакція на світло квола. З якого препарату необхідно розпочати надання невідкладної допомоги? A patient with epilepsy has constant large convulsive attacks with tonic and clonic convulsions, between which the patient remains unconscious. He does not respond to touch and painful stimuli. The pupils are narrow , reaction to light is weak. With which drug should emergency care be started?

Аміназин Aminazine

Ардуан Arduan

Барбовал Barboval

Димедрол Diphenhydramine

Сибазон Sibazone

95 / 200
У хворого 74-х років, на фоні аденоми простати, повторно виникла гостра затримка сечі. Катетеризація металевим катетером не мала успіху, з уретри виділяється кров. Пальпується напружений, переповнений сечовий міхур (дно - на 12 см вище лобка). Які дії лікаря? A 74-year-old patient, against the background of a prostate adenoma, has repeated acute urinary retention. Catheterization with a metal catheter was unsuccessful, blood is released from the urethra. Palpation is tense, an overflowing bladder (the bottom is 12 cm above the pubis). What are the doctor's actions?

Надлобкова пункція сечового міхура Suprapubic bladder puncture

Повторити катетеризацію сечового міхура металевим катетером Repeat bladder catheterization with a metal catheter

Негайна госпіталізація для проведення хірургічного втручання Immediate hospitalization for surgical intervention

- -

Катетеризація сечового міхура гумовим катетером Catheterization of the urinary bladder with a rubber catheter

96 / 200
У новонародженої дитини на третю добу життя з’явилися ознаки шлунково-кишкової кровотечі (блювання з домішками крові, мелена) та симптоми постгеморагічної анемії. З якого розчину слід розпочати парентеральне лікування геморагічного синдрому у дитини? On the third day of life, a newborn baby showed signs of gastrointestinal bleeding (vomiting with blood impurities, melena) and symptoms of posthemorrhagic anemia. Which solution should I start with parenteral treatment of hemorrhagic syndrome in a child?

Ілюкозо-сольовий розчин Ilucose-saline solution

Реополіглюкін Rheopoliglyukin

Тромбоцитарна маса Platelet mass

Свіжозаморожена плазма Fresh-frozen plasma

Амінокапронова кислота Aminocaproic acid

97 / 200
Хворий 60-ти років скаржиться на сильний біль у правій гомілці розпираючого характеру, підвищення температури тіла до 38oC. Об’єктивно: шкіра гомілки набрякла, гіперемована, під час пальпації різко болісна у ділянці гомілкових м’язів. Захворювання прогресує протягом 5 діб. Який найбільш вірогідний діагноз? A 60-year-old patient complains of severe pain in the right shin of a distending nature, an increase in body temperature to 38oC. Objectively: the skin of the shin is swollen, hyperemic, during palpation is sharply painful in the area of ​​the calf muscles. The disease progresses over 5 days. What is the most likely diagnosis?

Облітеруючий атеросклероз судин правої нижньої кінцівки Obliterating atherosclerosis of vessels of the right lower limb

Бешихове запалення правої гомілки Beshich's inflammation of the right lower leg

Післятромбофлебітичний синдром Postthrombophlebitic syndrome

Іострий флеботромбоз глибоких вен правої гомілки Acute phlebothrombosis of the deep veins of the right leg

Тромбофлебіт підшкірних вен правої гомілки Thrombophlebitis of the subcutaneous veins of the right leg

98 / 200
Хворий 40-ка років хворіє на цукровий діабет, отримує інсулін. Сьогодні ввечері посилилась загальна слабкість, з’явились спрага, поліурія. Напередодні відзначались сонливість, нудота, неодноразове блювання, біль у животі. Об’єктивно: непритомний. Сухість шкіри і слизових, зниження тургору шкіри і тонусу очних яблук. ЧД- 30/хв. Ps100 /хв., слабкого наповнення. Язик сухий, червоний. Запах ацетону з рота відсутній. Арефлексія. У крові: глюкоза - 42 ммоль/л, реакція на ацетон крові та сечі негативна. Яке дослідження необхідне провести для уточнення діагнозу в першу чергу? A 40-year-old patient suffers from diabetes, receives insulin. Tonight, general weakness increased, thirst, polyuria appeared. The day before, drowsiness, nausea, repeated vomiting, abdominal pain. Objectively: unconscious. Dry skin and mucous membranes, decreased skin turgor and tone of the eyeballs. ChD- 30/min. Ps100/min., weak filling. Tongue dry, red. The smell of acetone from the mouth is absent. Areflexia. In the blood: glucose - 42 mmol/l, the reaction to blood and urine acetone is negative. What research should be carried out to clarify the diagnosis in the first place?

Осмолярність крові Blood osmolarity

Білірубін крові Blood bilirubin

АлАТ, АсАТ AlAT, AsAT

Лужна фосфатаза Alkaline phosphatase

Креатинін крові Blood creatinine

99 / 200
Хлопчик 2-х років надійшов до стаціонару з вираженою папульозно-геморагічною висипкою на шкірі розгинальних поверхонь верхніх та нижніх кінцівок, сідницях. Висипка симетрична, має тенденцію до злиття. Спостерігається набряк та болючість великих суглобів, летючий біль. У крові: помірне підвищення кількості лейкоцитів, еозинофілія, помірна анемія. Який найбільш вірогідний діагноз? A 2-year-old boy was admitted to the hospital with a pronounced papular-hemorrhagic rash on the skin of the extensor surfaces of the upper and lower limbs, buttocks. The rash is symmetrical, has a tendency to merge. There is swelling and tenderness of large joints, flying pain. In the blood: a moderate increase in the number of leukocytes, eosinophilia, moderate anemia. What is the most likely diagnosis?

Менінгококцемія Meningococcemia

Гемофілія Hemophilia

Ієрсиніоз Yersiniosis

Ревматизм Rheumatism

Геморагічний васкуліт Hemorrhagic vasculitis

100 / 200
Хворий 30-ти років працює 8 років фасувальником анілінових барвників. Останню зміну працював у приміщенні, де вийшла з ладу вентиляція. Доставлений зі скаргами на головний біль, запаморочення, сонливість, різку загальну слабкість, пітливість. Об’єктивно: сірувато-синє забарвлення шкірних покривів, тони серця приглушені, Ps105 /хв., АТ-95/60 мм рт.ст., дихання поверхневе, ЧДР- 25/хв. Печінка +2 см, чутлива. У крові: 10% еритроцитів з тільцями Гейнца-Ерліха, лейк.- 8,5 • 109/л, ШЗЕ-2 мм/год. Які дослідження слід призначити для верифікації діагнозу? A 30-year-old patient has been working as a packer of aniline dyes for 8 years. He worked the last shift in a room where the ventilation failed. He was brought with complaints of headache, dizziness, drowsiness, sharp general weakness, sweating. Objectively: grayish-blue color of the skin, muffled heart sounds, Ps105/min., BP-95/60 mm Hg, shallow breathing, ChDR- 25/min. Liver + 2 cm, sensitive. In the blood: 10% erythrocytes with Heinz-Ehrlich bodies, leuk.- 8.5 • 109/l, SZE-2 mm/h. What tests should be prescribed to verify the diagnosis?

Аналіз сечі на свинець Urine analysis for lead

Аналіз крові на метгемоглобін Methemoglobin blood analysis

Аналіз сечі на метгемоглобін Urine analysis for methemoglobin

Аналіз крові на карбоксигемогло-бін Blood analysis for carboxyhemoglobin

Аналіз сечі на порфірини Analysis of urine for porphyrins

101 / 200
Вагітна у терміні 32 тижні страждає на міастенію. З’явились розлади дихання, тахікардія, психомоторне збудження, яке змінюється в’ялістю, парезом кишечнику та сфінктерів. Які першочергові засоби невідкладної допомоги? A 32-week pregnant woman suffers from myasthenia gravis. She has breathing disorders, tachycardia, psychomotor excitement, which is replaced by lethargy, paresis of the intestines and sphincters. What are the primary means of emergency aid?

Термінове розродження Urgent delivery

Застосування міорелаксантів Use of muscle relaxants

ШВЛ VENTILATOR

Призначення прозерину Proserin appointment

Профілактика гіпоксії плоду Prevention of fetal hypoxia

102 / 200
Хвора 44-х років скаржиться на біль в гомілці, озноб та підвищення температури до 39,3oC. На наступний день: гомілка набрякла, відзначається яскраве почервоніння шкіри. На стегні червоні 'доріжки'. Пахвинні лімфовузли не збільшені. Який найбільш імовірний діагноз? A 44-year-old patient complains of leg pain, chills, and an increase in temperature up to 39.3oC. The next day: the leg is swollen, bright reddening of the skin is noted. thighs red 'tracks'. Inguinal lymph nodes are not enlarged. What is the most likely diagnosis?

Бешихове запалення гомілки Beshikov inflammation of the lower leg

Інфікована рана гомілки Infected leg wound

Гострий тромбофлебіт підшкірних вен Acute thrombophlebitis of subcutaneous veins

Еризипелоїд гомілки Erysipeloid leg

Флегмона гомілки Phlegmon of the shin

103 / 200
Дитина народилася в машині швидкої допомоги на шляху до пологового будинку. У новонародженого відсутнє спонтанне дихання після погладжування шкіри уздовж хребта. Які подальші дії? The baby was born in the ambulance on the way to the maternity hospital. The newborn is not breathing spontaneously after stroking the skin along the spine. What are the next steps?

Поплескати по сідницях Pat on the buttocks

Дати кисень Give oxygen

Почати штучну вентиляцію легень Start CPR

Поплескати по п’яті Pat your heel

Повторювати погладжування Repeat swipe

104 / 200
Пацієнт 30-ти років хворіє на виразкову хворобу, ускладнену стенозом воротаря. Схуд на 15 кг за останні 2 місяці. Об’єктивно: кахексія, зниження тургору шкіри; в крові - анемія, гіпопротеїнемія, зниження концентрації електролітів. Яким шляхом доцільно здійснювати інфузійну терапію цьому пацієнту? A 30-year-old patient suffers from peptic ulcer disease, complicated by portal vein stenosis. He has lost weight by 15 kg over the past 2 months. Objectively: cachexia, decreased skin turgor; in the blood - anemia, hypoproteinemia, a decrease in the concentration of electrolytes. What is the appropriate way to administer infusion therapy to this patient?

Шляхом венесекції вен верхньої кінцівки By venesection of the veins of the upper limb

Через венепункцію Due to venipuncture

Через центральний венозний доступ Through central venous access

Вибір венозного доступу не має значення The choice of venous access does not matter

Шляхом венесекції вен нижньої кінцівки By venesection of the veins of the lower limb

105 / 200
У хворої на цукровий діабет іі типу розвинулась кома. Глікемія - 45 ммоль/л, калій - 4,4 ммоль/л, натрій - 158 ммоль/л, рН крові - 7,24, стандартний бікарбонат - 20 ммоль/л, зсув буферних основ - 4 ммоль/л, сечовина - 12 ммоль/л. Які засоби необхідно використати для регідратаційної терапії? A patient with type II diabetes developed a coma. Glycemia - 45 mmol/l, potassium - 4.4 mmol/l, sodium - 158 mmol/l, Blood pH - 7.24, standard bicarbonate - 20 mmol/l, shift of buffer bases - 4 mmol/l, urea - 12 mmol/l. What means should be used for rehydration therapy?

5% розчин глюкози 5% glucose solution

2,5% розчин глюкози 2.5% glucose solution

0,45% розчин натрію хлориду 0.45% sodium chloride solution

0,9% розчин натрію хлориду 0.9% sodium chloride solution

4% розчин натрію гідрокарбонату 4% sodium bicarbonate solution

106 / 200
У службовця протягом декількох годин після радіоактивного опромінення спостерігається лейкоцитоз (11,5 • 109/л) з переважанням нейтрофілів (сегменто- і паличкоядер-них) і зсувом вліво, лімфоцитопенія (0,1 — 1 • 109/л); в перші 2-3 доби - схильність до ретикулоцитозу. У кістковому мозку - зменшення кількості мі-єлокаріоцитів, еритробластів, числа мітозів, зниження мітотичного інде- ксу, зникнення молодих форм клітин, підвищення цитолізу. Про який період та ступінь важкості гострої променевої хвороби свідчать наведені показники? The employee has leukocytosis (11.5 • 109/l) with a predominance of neutrophils (segment- and rod-nuclear) and a shift to the left within a few hours after radioactive exposure , lymphocytopenia (0.1 — 1 • 109/l); in the first 2-3 days - a tendency to reticulocytosis. In the bone marrow - a decrease in the number of myelokaryocytes, erythroblasts, the number of mitoses, a decrease in the mitotic index, the disappearance of young cell forms , increased cytolysis. What period and degree of severity of acute radiation sickness do the above indicators indicate?

Прихований період, легкий ступінь Hidden period, easy degree

Період віддалених наслідків, середній ступінь Period of remote consequences, medium degree

Період виходів, важкий ступінь Period of withdrawals, severe degree

Період розпалу, вкрай важкий ступінь Period of fever, extremely severe degree

Період загальної первинної реакції, середній ступінь Period of total primary reaction, average degree

107 / 200
Хворий 72-х років доставлений машиною швидкої допомоги у важкому стані із запамороченням. На ЕКГ: передсердні та шлуночкові комплекси виникають незалежно, кількість шлуночкових комплексів 36/хв. Дифузні зміни міокарда. Які препарати слід призначити першочергово? A 72-year-old patient was brought by ambulance in a serious condition with dizziness. On the ECG: atrial and ventricular complexes occur independently, the number of ventricular complexes is 36/min. Diffuse changes in the myocardium. What drugs should be prescribed first?

Строфантин Strophantin

Препарати калію Potassium drugs

Анаприлін Anaprilin

Ізадрин, атропін Izadrine, atropine

Адреналін Adrenaline

108 / 200
Дитина 4-х років скаржиться на блювання до 10 разів на добу після того, як поїла копчену рибу, значну спрагу, порушення зору у вигляді подвоєння. Об’єктивно: мідріаз, горизонтальний ністагм, птоз, поперху-вання. Який найбільш імовірний діагноз? A 4-year-old child complains of vomiting up to 10 times a day after eating smoked fish, great thirst, impaired vision in the form of doubling. Objectively : mydriasis, horizontal nystagmus, ptosis, fluttering. What is the most likely diagnosis?

Ангіна Angina

Ботулізм Botulism

Правець Tetanus

Сказ Rabies

Дифтерія Diphtheria

109 / 200
Перебіг післяопераційного періоду у хворого ускладнився виникненням делірійного синдрому. З анамнезу відомо, що пацієнт тривалий час зловживає алкоголем. Препаратом вибору для корекції психомоторного збудження в даному випадку є: The course of the patient's postoperative period was complicated by the occurrence of delirium syndrome. It is known from the anamnesis that the patient has been abusing alcohol for a long time. The drug of choice for the correction of psychomotor agitation in this case is:

Етанол Ethanol

Аміназин Aminazine

Пропофол Propofol

Сибазон Sibazone

Галоперидол Haloperidol

110 / 200
У хворого 62-х років, що знаходиться на лікуванні в кардіології з приводу загострення ІХС і страждає 10 років на цукровий діабет, настала непритомність після ін’єкції інсуліну. Яка причина гіпоглікемічної коми? A 62-year-old patient, who is being treated in cardiology for an exacerbation of coronary artery disease and has been suffering from diabetes for 10 years, fainted after an insulin injection. What is the cause of hypoglycemic coma?

Надмірне введення рідини в організм Excessive introduction of fluid into the body

Недостатня доза інсуліну Insufficient dose of insulin

Відсутність прийому їжі після ін’єкції інсуліну Absence of eating after insulin injection

Надлишкова кількість введеного інсуліну Excess amount of injected insulin

Переїдання Overeating

111 / 200
Чоловік 45-ти років звернувся до сімейного лікаря зі скаргами на стискаючий біль за грудниною, що з’являється під час швидкої ходи, тривалістю 3-5 хвилин. Під час реєстрації ЕКГ знепритомнів, розвився виражений ціаноз, пульс на а.cаrotіs не визначається, зіниці широкі, на світло не реагують. На ЕКГ: спочатку зареєстровано синусовий ритм, ЧСС-85/хв., сегменти ST зміщено на 4 мм догори від ізоелектричної лінії, потім з’являються хаотичні неправильної форми хвилі. Назвіть причину клінічної смерті: A 45-year-old man turned to his family doctor with complaints of squeezing pain behind the sternum that appears during fast walking and lasts 3-5 minutes. Under during the recording of the ECG, he fainted, severe cyanosis developed, the pulse on the a.carotis was not determined, the pupils were wide, they did not react to light. On the ECG: initially sinus rhythm was registered, heart rate-85/min., ST segments were shifted 4 mm above the isoelectric line , then chaotic irregular waveforms appear. Name the cause of clinical death:

Повна атріовентрикулярна блокада Complete atrioventricular block

Миготіння передсердь Atrial fibrillation

Тромбоемболія легеневої артерії Thromboembolism of the pulmonary artery

Асистолія Asystole

Фібриляція шлуночків Ventricular fibrillation

112 / 200
Хвора 44-х років перенесла операцію субтотальної резекції щитоподібної залози з приводу дифузного токсичного зобу. На другу добу стан хворої погіршився: з’явилося серцебиття, задишка, пітливість, пронос, почуття страху. Об’єктивно: хвора збуджена, шкіра волога, гаряча на дотик. Температура тіла 39,2oC. Тони серця приглушені, тахікардія. Ps-160/хв, АТ- 85/40 мм рт.ст. Живіт м’який, безболісний. Які дослідження потрібно провести для встановлення діагнозу? A 44-year-old patient underwent subtotal resection of the thyroid gland due to a diffuse toxic goiter. On the second day, the patient's condition worsened: palpitations, shortness of breath, sweating, diarrhea, a sense of fear. Objectively: the patient is excited, the skin is moist, hot to the touch. Body temperature is 39.2oC. Heart sounds are muffled, tachycardia. Ps-160/min, BP- 85/40 mmHg. Abdomen m' which, painless. What tests need to be done to establish a diagnosis?

Вміст тиреоїдних гормонів Thyroid hormone content

Вміст адреналіну та норадреналіну в крові Content of adrenaline and norepinephrine in the blood

Вміст 17-КС, 17-ОКС в сечі Content of 17-KS, 17-OKS in urine

Вміст трансаміназ сироватки крові Content of transaminases in blood serum

Вміст цукру крові Blood sugar content

113 / 200
Дитина 1-го року доставлена до лікарні з діагнозом: гостра ниркова недостатність, стадія олігоанурії. У хворого відзначається м’язова слабкість, аритмія, зміни на ЕКГ ( розширення інтервалів PQ та QRS, високі, вузькі та симетричні зубці T). Які біохімічні зміни викликають ці порушення? A 1-year-old child was brought to the hospital with a diagnosis of acute renal failure, stage of oligoanuria. The patient has muscle weakness, arrhythmia, changes in the ECG (extension PQ and QRS intervals, tall, narrow and symmetrical T-waves). What biochemical changes cause these disturbances?

Гіперкаліємія Hyperkalemia

Гіперкальціємія Hypercalcemia

Гіпокальціємія Hypocalcemia

Гіпокаліємія Hypokalemia

Ацидоз Acidosis

114 / 200
Будівельник 27-ми років під час виконання робіт був притиснутий до стіни будинку бортом автомобіля. Скаржиться на біль у ділянці таза, неможливість підняти прямі нижні кінцівки, слабкість, запаморочення. Який додатковий метод діагностики є оптимальним? A 27-year-old builder was pressed against the wall of a house by the side of a car during work. He complains of pain in the pelvis, inability to raise straight lower limbs, weakness, dizziness . What additional diagnostic method is optimal?

Оглядова рентгенографія таза Comprehensive radiography of the pelvis

ЯМРТ, КТ, оглядова рентгенографія таза NMR, CT, X-ray of the pelvis

Рентгеноскопія таза X-ray pelvis

КТ таза Pelvic CT

ЯМРТ таза MRI pelvis

115 / 200
Під час гемотрансфузії у дитини з кровотечею виникли біль у грудях, попереку, з’явилася задишка; сеча набула червоного забарвлення. Плазма крові після центрифугування рожева. Який найбільш імовірний діагноз? During hemotransfusion, a child with bleeding developed pain in the chest, lower back, shortness of breath; the urine became red. The blood plasma after centrifugation is pink. What is the most likely diagnosis?

Гострий гемоліз Acute hemolysis

Негемолітична трансфузійна реакція Nonhemolytic transfusion reaction

Пірогенна реакція Pyrogenic reaction

Синдром масивних трансфузій Massive transfusion syndrome

Алергічна реакція Allergic reaction

116 / 200
На місці автопригоди у хворого була відірвана кисть. Які дії при консервації видаленого сегмента? At the scene of the car accident, the patient's hand was torn off. What actions are taken to preserve the removed segment?

Помістити в холодну воду Place in cold water

Помістити в холод Put in the cold

Прифіксувати кисть до передпліччя 117. У вагітної у терміні 37 тижнів відбувся напад судом. АТ- 160/100 та 170/100 мм рт.ст. Fix the brush to the forearm 117. A pregnant woman had a convulsion at 37 weeks. BP - 160/100 and 170/100 mm Hg.

Провести ПХО сегмента Perform PHO segment

Надати симптоматичну допомогу Provide symptomatic care

117 / 200
Набряки нижніх кінцівок, живота, обличчя. Дихання відсутнє, шкірні покриви ціанотичні. Що потрібно зробити у першу чергу? Swelling of the lower limbs, abdomen, face. There is no breathing, the skin is cyanotic. What should be done first?

Забезпечити прохідність дихальних шляхів Ensure patency of airways

Ендотрахеальний наркоз Endotracheal anesthesia

Внутрішньовенне введення проти-судомних препаратів Intravenous administration of anticonvulsants

Внутрішньовенне введення гіпотензивних препаратів Intravenous administration of hypotensive drugs

Внутрішньовенне введення седативних препаратів Intravenous administration of sedatives

118 / 200
Хворий 42-х років скаржиться на часті дефекації (4-8 на добу) з кров’ю (від прожилок до яскраво-червоної крові), із слизом та гноєм. Температура тіла 37,2oC-37,5oC. Відмічає ано-рексію, схуднення, швидку втомлюваність. В крові: анемія. Ендоскопічно: слизова оболонка товстої кишки малиново-червоного кольору, з мікроабсцесами, з виразками, вкритими фібрином, гноєм; при дотику -кровить. Який найбільш імовірний діагноз? A 42-year-old patient complains of frequent defecation (4-8 per day) with blood (from veins to bright red blood), with mucus and with pus. Body temperature 37.2oC-37.5oC. Anorexia, weight loss, rapid fatigue are noted. In the blood: anemia. Endoscopically: the mucous membrane of the large intestine is crimson-red in color, with microabscesses, with ulcers covered with fibrin, pus; with touch - bleeds. What is the most likely diagnosis?

Неспецифічний виразковий коліт Nonspecific ulcerative colitis

Туберкульозний ілеотифлін Tuberculous ileotiflin

Рак товстої кишки Colon cancer

Поліпоз товстої кишки Colon polyposis

Хвороба Крона Crohn's disease

119 / 200
У жінки в першу добу післяопераційного періоду після екстирпації матки, яка була виконана в ургентному порядку у зв’язку із кровотечею -анурія. Яке обстеження необхідно виконати для встановлення діагнозу? A woman on the first day of the postoperative period after extirpation of the uterus, which was performed urgently due to bleeding, has anuria. What examination must be performed to establish a diagnosis ?

Реовазографія нирок Renal rheovasography

Ультразвукове дослідження нирок Ultrasound study of kidneys

Екскреторна урографія Excretory urography

Оцінка кровотоку нирок за допомогою доплерометрії Estimation of kidney blood flow using dopplerometry

Дослідження сироватки крові на вміст креатиніну Blood serum test for creatinine content

120 / 200
Хворий 37-ми років доставлений до лікарні в непритомному стані. До цього біля 3-х годин працював з газозварювальним апаратом у закритому приміщенні. Об’єктивно: шкіра та слизові рожевого кольору; зіниці розширені, відсутня їх реакція на світло; поява тонічних та клонічних судом. ЧД- 38/хв., Ps- 116/хв., АТ- 95/60 мм рт.ст., тони серця ослаблені. Який з додаткових методів дослідження дозволяє підтвердити діагноз? A 37-year-old patient was brought to the hospital in an unconscious state. Before that, he worked with a gas welding machine in a closed room for about 3 hours. Objectively: the skin and mucous membranes are pink in color; the pupils are dilated, there is no reaction to light; the appearance of tonic and clonic convulsions. BH - 38/min., Ps - 116/min., BP - 95/60 mm Hg, heart tones are weakened. Which of the following additional research methods allow to confirm the diagnosis?

Визначення в крові вмісту метгемоглобіну Determination of methemoglobin content in blood

Визначення в еритроцитах тілєць Іейнца-Ерліха Determination of Einz-Ehrlich bodies in erythrocytes

Ехоенцефалографія Echoencephalography

Визначення в крові вмісту карбо-ксигемоглобіну Determination of carbo-xyhemoglobin content in blood

ЕКГ ECG

121 / 200
Вранці мама знайшла 5-місячного хлопчика в ліжечку вже мертвим з трупними плямами, обличчям донизу. Перед цим дитина була здоровою. Яка найбільш імовірна причина смерті дитини? In the morning, the mother found a 5-month-old boy in the crib already dead with corpse stains, face down. Before that, the child was healthy. What is the most likely cause of the child's death?

Кардіогенний шок Cardiogenic shock

Аспірація блювотними масами Aspiration of vomitus

Синдром раптової смерті Sudden Death Syndrome

Епілептичний статус Status Epileptic

Менінгоенцефаліт Meningoencephalitis

122 / 200
У дитини 8-ми місяців на 4-й день стаціонарного лікування з приводу гострої кишкової інфекції різко погіршився стан. Відмічається виражена блідість з лимонно-жовтим відтінком шкіри, петехіальний висип, мелена, анурія. У крові анемія, ретикуло-цитоз, тромбоцитопенія, лейкоцитоз. Про розвиток якого стану можна думати в даному випадку? On the 4th day of inpatient treatment for an acute intestinal infection, an 8-month-old child's condition worsened. Marked pallor with a lemon-yellow skin tone, petechial rash, melena, anuria. Anemia, reticulocytosis, thrombocytopenia, leukocytosis in the blood. What kind of condition can we think about in this case?

Гіпопластична анемія Hypoplastic anemia

Гостра ниркова недостатність Acute renal failure

Гемолітична анемія Hemolytic anemia

Гемолітико-уремічний синдром Hemolytic uremic syndrome

Лейкоз Leukemia

123 / 200
Жінка 75-ти років внаслідок бічного удару легковика впала з висоти власного зросту на лівий бік. Відчула різкий біль у лівому кульшовому суглобі, хрускіт, самостійно підвестися не змогла. Який діагноз найбільш імовірний? A 75-year-old woman fell from a height of her own height onto her left side as a result of a side impact from a car. She felt a sharp pain in her left hip joint, a crunch, and could not get up on her own. What is the most likely diagnosis?

Відкритий перелом шийки правої стегнової кістки зі зміщенням Open fracture of the neck of the right femur with displacement

Закритий перелом шийки правої стегнової кістки зі зміщенням Closed fracture of the neck of the right femur with displacement

Забій лівого кульшового суглоба Left Hip Contusion

Відкритий перелом шийки лівої стегнової кістки зі зміщенням Open fracture of the neck of the left femur with displacement

Закритий перелом шийки лівої стегнової кістки зі зміщенням Closed fracture of the neck of the left femur with displacement

124 / 200
У хворого 59-ти років, що знаходиться в стаціонарі з приводу ІХС, діагностовано раптову зупинку сер- цевої діяльності та дихання, почато легенево-серцеву реанімацію. На ЕКГ: тріпотіння шлуночків. Для відновлення ритму потрібно: A 59-year-old patient, who is in the hospital due to coronary artery disease, was diagnosed with sudden cardiac arrest and breathing, cardiopulmonary resuscitation was started. On the ECG : fluttering of the ventricles. To restore the rhythm, you need:

Масаж очних яблук Eyeball massage

Удари в міжлопаткову ділянку Blows in the interscapular area

Прекардіальний удар Precardiac attack

Масаж області каротидного синуса Massage of the carotid sinus area

Перкусійний масаж Percussion massage

125 / 200
Пацієнтка 69-ти років вночі, напередодні госпіталізації, відчувала стан тривоги. Їй здавалося, що в дім хтось намагається проникнути, була розгальмована. На цьому фоні виникли тоніко-клонічні напади зі світлим проміжком до 3 хвилин. Стан слід кваліфікувати як: The 69-year-old patient felt a state of anxiety at night, on the eve of hospitalization. She thought that someone was trying to break into the house, she was disinhibited. Against this background, tonic- clonic seizures with a light interval of up to 3 minutes. The condition should be classified as:

Епілептичний статус Status epilepticus

Геморагічний інсульт Hemorrhagic stroke

Немічний інсульт Debilitating stroke

Наслідки ЧМТ Consequences of TBI

Отруєння наркотиками Drug poisoning

126 / 200
Новонароджена дитина в терміні гестації 39 тижнів з масою 3500 г, довжиною 54 см. При народженні - асфіксія середнього ступеня важкості. Оцінка за шкалою Апгар на 1-й хвилині 4 бали. Після проведення первинної реанімаційної допомоги з’явилося самостійне дихання, ЧСС-110/хв., спостерігається акроціаноз. Яка подальша дія лікарянеонатолога у даній ситуації? A newborn baby at 39 weeks' gestation with a weight of 3500 g, a length of 54 cm. At birth - asphyxia of an average degree of severity. Apgar score at the 1st minute 4 points. After primary resuscitation, spontaneous breathing appeared, heart rate 110/min., acrocyanosis is observed. What is the further action of the neonatologist in this situation?

Додаткова оксигенація Additional oxygenation

Штучний масаж серця Artificial heart massage

Допоміжна вентиляція легень Assisted lung ventilation

Інтубація трахеї Tracheal intubation

Відсмоктування слизу з верхніх дихальних шляхів Suction of mucus from the upper respiratory tract

127 / 200
Хворий 72-х років знаходиться на стаціонарному лікуванні зі скаргами на інтенсивний біль голови, підвищення температури тіла до 38,5oC. Об’єктивно: на шкірі тулуба й кінцівок розеольозно-петехіальна висипка. Встановлено, що в дитинстві пацієнт перебував у вогнищі висипного тифу. Для підтвердження діагнозу необхідно провести: A 72-year-old patient is undergoing inpatient treatment with complaints of an intense headache, an increase in body temperature up to 38.5oC. Objectively: on the skin of the trunk and limbs roseolous-petechial rash. It has been established that the patient was in an outbreak of typhus as a child. To confirm the diagnosis, it is necessary to conduct:

Серологічні дослідження Serological studies

Бактеріологічне дослідження крові Bacterial blood test

Мікроскопічне дослідження зіскобу розеол Microscopic study of roseola scraping

Бактеріологічне дослідження зіско-бу розеол Bacteriological study of roseola

Мікроскопічне дослідження крові Microscopic examination of blood

128 / 200
До приймального відділення надійшов хворий з підозрою на наявність стороннього тіла в ротоглотці: напад інспіраторної задишки, яка супроводжується сильним кашлем, голос оси-плий, болі в горлі та за грудниною. Прийом Іеймліха неефективний. Видалити стороннє тіло при прямій ларингоскопії неможливо. Стан хворого погіршується. Який першочерговий захід для надання допомоги? A patient was admitted to the reception department with a suspicion of the presence of a foreign body in the oropharynx: an attack of inspiratory dyspnea accompanied by a strong cough, a wheezing voice, pain in the throat and by the sternum. Ieimlich's reception is ineffective. It is impossible to remove a foreign body during direct laryngoscopy. The patient's condition is worsening. What is the primary measure to provide assistance?

Конікотомія Conicotomy

Трахеотомія Tracheotomy

Потрійний прийом Сафара Triple reception of Safar

Інтубація трахеї з переведенням на ШВЛ Tracheal intubation with transfer to ventilator

Проштовхнути стороннє тіло вглиб Push foreign body deep

129 / 200
Хворій 25-ти років після виконання операції з приводу порушеної трубної вагітності проводиться переливання еритроцитарної маси та сольових розчинів. Об’єктивно: АТ100/50 мм рт.ст., Ps- 92/хв., ЧД- 16/хв. У крові: Hb- 84 г/л, Ht- 31%, ер.- 2,3 ■ 1012/л. Раптово хвора вказує на появу головного болю, ядухи, болю за грудниною та у попереку. З чим пов’язана поява цих симптомів? A 25-year-old patient undergoes a transfusion of erythrocyte mass and saline solutions after an operation for a broken tubal pregnancy. Objectively: blood pressure 100/50 mmHg. , Ps- 92/min., ChD- 16/min. In the blood: Hb- 84 g/l, Ht- 31%, er.- 2.3 ■ 1012/l. Suddenly the patient indicates the onset of headache, nausea, pain behind the sternum and in the lower back. What is associated with the appearance of these symptoms?

Незупинена кровотеча Continuous bleeding

Неврологічні порушення Neurological disorders

Постнаркозні порушення Post-narcotic disorders

Алергічна реакція Allergic reaction

Несумісність крові Blood incompatibility

130 / 200
Вантажник судна, що повернулося з круїзу в Середземному морі, звернувся до хірурга у зв’язку з погіршенням стану рани після укусу щура в трюмі судна. Відзначає слабкість, головний біль, to- 40oC. Об’єктивно: нога набрякла, збільшені пахвинні лімфовузли з одного боку, 3-4 см в діаметрі. АТ- 90/60 мм рт.ст., Ps- 100/хв. Запідозрено чуму, бубонний варіант. Які першочергові заходи? A freighter on a ship returning from a cruise in the Mediterranean sought a surgeon due to a worsening wound after being bitten by a rat in the ship's hold. Noting weakness, the chief pain, to- 40oC. Objectively: the leg is swollen, enlarged inguinal lymph nodes on one side, 3-4 cm in diameter. BP- 90/60 mm Hg, Ps- 100/min. Plague is suspected, bubonic variant. What are the priority measures?

Обробка рани, введення протистол-бнячної сироватки Treatment of the wound, administration of antitumor serum

Іоспіталізація в терапевтичне відділення Hospitalization in the therapeutic department

Введення дексаметазону, кордіаміну, госпіталізація в реанімаційне відділення Introduction of dexamethasone, cordiamine, hospitalization in the intensive care unit

Іоспіталізація в інфекційне відділення Hospitalization in the infectious department

Іоспіталізація в хірургічне відділення Hospitalization in the surgical department

131 / 200
В селищі зареєстровано випадок сибірки у місцевого жителя, який заразився під час вибілювання туші хворої на сибірку корови. М’ясо тварини було розподілено між сім’ями родичів хворого для власного споживання. Оцініть санітарно-епідемічний стан селища: In the village, a case of anthrax was registered in a local resident who became infected while bleaching the carcass of a cow with anthrax. The meat of the animal was distributed among the families of the patient's relatives for own consumption. Assess the sanitary-epidemic condition of the village:

Незадовільний Unsatisfactory

Благополучний Prosperous

Надзвичайний Extraordinary

Нестійкий Unstable

Неблагополучний Disadvantaged

132 / 200
Чоловік 80-ти років доставлений до приймального відділення після еле-ктротравми. Непритомний, пульс та дихання не визначаються. Які необхідні дії? An 80-year-old man was brought to the emergency room after electrocution. He is unconscious, pulse and breathing are not detected. What are the necessary actions?

Дезінтоксикаційна терапія Detoxification therapy

Серцево-легенева реанімація Cardiopulmonary resuscitation

Іоспіталізація до відділення інтенсивної терапії Hospitalization to the intensive care unit

Штучне дихання Artificial respiration

Стерильна пов’язка на місце ураження Sterile dressing on the affected area

133 / 200
У пацієнтів з важкою емфіземою легень трахеостомія дозволить в першу чергу: In patients with severe pulmonary emphysema, tracheostomy will allow first of all:

Попередити набряк легень Prevent pulmonary edema

Зволожити повітря Humidify the air

Викликати задишку Induce shortness of breath

Зменшити мертвий простір Reduce dead space

Видалити секрет Delete secret

134 / 200
При променевому ураженні ттткі-ри у хворого спостерігається вологий дерматит, пухирі малі, ненапружені, займають менш ніж 50% поля. Така клінічна картина виникає при дозі опромінення: In the case of radiation damage to the tttki-ra, the patient has moist dermatitis, the blisters are small, non-tense, occupying less than 50% of the field. The following clinical picture occurs with the radiation dose:

15-20 Гр. 15-20 Gy.

5-8 Гр. 5-8 Gy.

20-30 Гр. 20-30 Gy.

7-13 Гр. 7-13 Gy.

10-15 Гр. 10-15 Gy.

135 / 200
Вагітна 40 тижнів з бурхливою пологовою діяльністю, клінічно вузьким тазом. Під час індукції наркозу виникло блювання. Після інтуба-ції проведена санація дихальних шляхів та ротової порожнини. В подальшому розвинувся ціаноз, підвищився центральний венозний тиск, над легенями різнокаліберні вологі хрипи, підвищення тиску на вдиху. При контролі газів крові - значна гіпоксемія. Яка причина цього стану? 40 weeks pregnant with violent labor, clinically narrow pelvis. Vomiting occurred during the induction of anesthesia. After intubation, sanitation of the respiratory tract and oral cavity was carried out. Later cyanosis has developed, central venous pressure has increased, wet rales of various calibers over the lungs, increased inspiratory pressure. When blood gases are monitored, there is significant hypoxemia. What is the cause of this condition?

Ендотоксичний шок Endotoxic shock

Шфаркт міокарда Myocardial infarction

Емболія навколоплідними водами Amniotic fluid embolism

Синдром Мендельсону Mendelssohn syndrome

Набряк легенів Pulmonary edema

136 / 200
Дитина від третьої вагітності, других пологів, термін гестації 29 тижнів, маса тіла 1050 г, довжина 43 см. При народженні реакція на огляд відсутня, дифузний ціаноз, дихання по типу 'гаспінг'. чСС- 120/хв. Визначте патогенетичну терапію: Child from third pregnancy, second birth, gestation period 29 weeks, body weight 1050 g, length 43 cm. At birth, there is no reaction to examination, diffuse cyanosis, breathing according to the 'gasping' type. Heart rate - 120/min. Define pathogenetic therapy:

Провести тактильну стимуляцію дихання Perform tactile breathing stimulation

Дати 100% кисень та ввести простагландин Е Give 100% oxygen and administer prostaglandin E

Заінтубувати та почати ШВЛ Intubate and start ventilator

Заінтубувати дитину, ввести штучний сурфактант Intubate the child, introduce artificial surfactant

Розпочати ШВЛ за допомогою маски Start ventilator with mask

137 / 200
Хворого 56-ти років госпіталізовано до хірургічного відділення з діагнозом - цироз печінки, кровотеча з вен стравоходу, асцит, гостра печінкова недостатність ііі ступеня. Яку лікувальну тактику слід обрати в цьому випадку? A 56-year-old patient was hospitalized in the surgical department with a diagnosis of cirrhosis of the liver, bleeding from the veins of the esophagus, ascites, acute liver failure of the 3rd degree. What treatment tactics should be chosen in this case?

Виконати сплено-ренальний анастомоз Perform spleno-renal anastomosis

Виконати прошивання вен стравоходу Perform esophageal vein stitching

Виконати гастростомію Perform gastrostomy

Виконати проксимальну резекцію шлунка Perform proximal gastric resection

Консервативна гемостатична терапія в умовах відділення, ввести зонд Блекмора Conservative hemostatic therapy in the department, insert a Blackmore probe

138 / 200
У хворого у відділенні реанімації зафіксована зупинка ефективного кровообігу. Який з нижче перерахованих препаратів при внутрішньовенному введенні забезпечить ефективну реанімацію? A patient in the intensive care unit has stopped effective blood circulation. Which of the drugs listed below will provide effective resuscitation when administered intravenously?

Внутрішньовенно адреналін по 1 мг кожні 5 хвилин реанімації Intravenous adrenaline 1 mg every 5 minutes of resuscitation

Налагодити введення реосорбіла-кту Debug reosorbil-ktu input

Першочергово необхідно ввести бікарбонат натрію 200 мл 4,2% розчину First of all, it is necessary to introduce sodium bicarbonate 200 ml of 4.2% solution

Негайно розпочати внутрішньовенну інфузію глюкози Start intravenous glucose infusion immediately

Використати гіпертонічний розчин хлориду натрію Use hypertonic sodium chloride solution

139 / 200
Хворому 19-ти років з наявністю в анамнезі підвищеної кровоточивості слизових оболонок рота, носових кровотеч планується проведення екстракції зуба. Об’єктивно: стан задовільний. Шкіра блідо-рожева. Дихання везикулярне. АТ- 120/70 мм рт.ст. Ps- 72/хв., задовільних властивостей. Використання яких лікарських засобів повинно передувати проведенню цієї стоматологічної маніпуляції? A 19-year-old patient with a history of increased bleeding of the mucous membranes of the mouth, epistaxis is scheduled for tooth extraction. Objectively: the condition is satisfactory. The skin is pale pink . Breathing is vesicular. Blood pressure - 120/70 mm Hg. Ps - 72/min., satisfactory properties. What medicines should be used before this dental manipulation?

Амінокапронова кислота 5% 100 мл в/в за 30 хвилин до екстракції зуба Aminocaproic acid 5% 100 ml IV 30 minutes before tooth extraction

Ліофілізований концентрат фактору Vііі20 ОД/кг до екстракції зуба Lyophilised concentrate of factor VIII 20 units/kg before tooth extraction

Фібриноген 3 г в/в за 30 хвилин до екстракції зуба Fibrinogen 3 g IV 30 minutes before tooth extraction

Вікасол в/м за 30 хвилин до екстракції зуба Vikasol IV 30 minutes before tooth extraction

Етамзилат натрію в/в за 30 хвилин до екстракції зуба Etamsylate sodium IV 30 minutes before tooth extraction

140 / 200
Жінку 68-ми років, яка хворіє на цукровий діабет іі типу (супутня патологія - хронічний пієлонефрит, хронічна ниркова недостатність), госпіталізовано до реанімаційного відділення з приводу лактоацидотичної коми. Об’єктивно: свідомість відсутня, шкіра бліда, дихання Кусмауля, АТ- 95/70 мм рт.ст., ЧСС- 100/хв. Глікемія 9,2 ммоль/л. Яку схему інсуліноте-рапії найбільш доцільно використати у хворої? A 68-year-old woman suffering from type II diabetes (combined pathology - chronic pyelonephritis, chronic renal failure) was hospitalized in the intensive care unit due to lactic acidosis coma Objectively: consciousness is absent, the skin is pale, Kussmaul's breathing, blood pressure - 95/70 mm Hg, heart rate - 100/min, blood glucose 9.2 mmol/l. What scheme of insulin therapy is most appropriate to use in the patient?

12-14 ОД пролонгованого інсуліну на добу на тлі внутрішньовенної інфу-зії 5% розчину глюкози з розрахунку 1 ОД інсуліну на 4 г глюкози 12-14 units of prolonged insulin per day against the background of intravenous infusion of 5% glucose solution at the rate of 1 unit of insulin per 4 g of glucose

12-14 ОД пролонгованого інсуліну на добу 12-14 units of long-acting insulin per day

0,15 ОД/кг/год простого інсуліну, всього -12 годин 0.15 U/kg/hour simple insulin, only -12 hours

0,1 ОД/кг/год простого інсуліну впродовж доби 0.1 U/kg/h simple insulin during the day

2-4 ОД простого інсуліну на годину на тлі внутрішньовенної інфузії 5% розчину глюкози з розрахунку 1 ОД інсуліну на 4 г глюкози 2-4 units of simple insulin per hour against the background of intravenous infusion of 5% glucose solution at the rate of 1 unit of insulin per 4 g of glucose

141 / 200
Пологи закінчилися накладенням порожнинних акушерських щипців і народженням дитини в гіпоксії тяжкого ступеня. Неонатолог у пологовому залі провів реанімаційні заходи і взяв новонародженого на ШВЛ. Де краще проводити лікування цієї дитини? The delivery ended with the application of cavity obstetric forceps and the birth of a child in severe hypoxia. The neonatologist in the delivery room performed resuscitation measures and took the newborn to a ventilator. Where is the best place to treat this child ?

У пологовому залі In the delivery room

У нейрохірургічному відділенні дитячої лікарні In the neurosurgical department of a children's hospital

У відділенні новонароджених In the neonatal unit

У неврологічному відділенні In the neurology department

У відділенні реанімації спеціалізованої дитячої лікарні In the intensive care unit of a specialized children's hospital

142 / 200
Пацієнт 47-ми років прокинувся вночі від різкого болю в плюсно-фаланговому суглобі першого пальця правої стопи, підвищення температури тіла до 38oC. Напередодні ввечері після тривалого фізичного навантаження випив 0,5 л пива. Об’єктивно: дефігурація суглоба, шкіра над ним гі-перемована з ціанотичним відтінком, гаряча на дотик, різка болючість під час пальпації. Яке обстеження необхідно провести для підтвердження діагнозу? A 47-year-old patient woke up at night from a sharp pain in the metatarsophalangeal joint of the first toe of the right foot, an increase in body temperature to 38oC. The night before, after prolonged physical exertion, he drank 0.5 l of beer. Objectively: joint deformity, the skin over it is hyperpigmented with a cyanotic shade, hot to the touch, sharp pain during palpation. What examination should be performed to confirm the diagnosis?

Визначити рівень гемоглобіну в крові Determine the level of hemoglobin in the blood

Визначити рівень глюкози в крові Determine blood glucose level

Визначити рівень лужної фосфатази Determine the level of alkaline phosphatase

Визначити рівень кальцію в крові Determine the level of calcium in the blood

Визначити рівень сечової кислоти в крові Determine the level of uric acid in the blood

143 / 200
У зоні відповідальності багато- профільної клінічної лікарні відбувся акт масового біотероризму. У найближчий час очікується прибуття великої кількості постраждалих, а також звернення осіб, що підозрюють наявність симптомів отруєння, внаслідок швидкого поширення інформації про терористичний акт у ЗМІ. Тип патогенного чинника ще невідомий. За якими факторами головний лікар буде організовувати сортування пацієнтів? An act of mass bioterrorism took place in the area of ​​responsibility of a multi-specialty clinical hospital. In the near future, a large number of victims are expected to arrive, as well as applications from persons who suspect the presence of symptoms of poisoning, due to the rapid spread of information about a terrorist act in the mass media. The type of pathogenic factor is still unknown. By what factors will the chief physician organize the sorting of patients?

Згідно з параметрами гемодинаміки According to hemodynamic parameters

Згідно з віком пацієнта According to the patient's age

За фізіологічними ознаками, анатомічною локалізацією ураження By physiological signs, anatomical localization of the lesion

За тривалістю після імовірного ураження By duration after probable damage

Сортування не відбуватиметься до отримання точнішої інформації про тип збудника Sorting will not occur until more accurate information about the pathogen type is obtained

144 / 200
Жінка 75-ти років страждає на гіпертонічну хворобу, миготливу аритмію. В анамнезі інфаркт міокарда 2 роки тому. Знаходиться на стаціонарному лікуванні з приводу раку шкіри. Раптово знепритомніла, пульс та артеріальний тиск не визначаються. На ЕКГ крупнохвильова фібриляція шлуночків. Що першочергово необхідно зробити? A 75-year-old woman suffers from hypertension, atrial fibrillation. She has a history of myocardial infarction 2 years ago. She is undergoing inpatient treatment for skin cancer. She suddenly fainted, the pulse and blood pressure are not determined. On the ECG, there is large-wave ventricular fibrillation. What should be done as a matter of priority?

Дефібриляція Defibrillation

Преднізолон Prednisone

Адреналін внутрішньовенно Adrenaline intravenously

Інтубація трахеї та ШВЛ Tracheal intubation and ventilator

Дофамін 2 мкг/кг/хв Dopamine 2 mcg/kg/min

145 / 200
У хворого 40-ка років на фоні фізичного напруження з’явився інтенсивний біль у правій скроневій ділянці. Об’єктивно: стан хворого важкий. Виражене оглушення, періодично збуджений. АТ- 170/100 мм рт.ст. Ригідність м’язів потилиці. Симптоми вогнищевого ураження нервової системи не виявлено. Який найбільш імовірний діагноз? A 40-year-old patient developed intense pain in the right temporal region against the background of physical exertion. Objectively: the patient's condition is severe. Pronounced stupor, periodically excited BP - 170/100 mm Hg. Stiffness of the muscles of the back of the head. No symptoms of focal damage to the nervous system were detected. What is the most likely diagnosis?

Гостра гіпертонічна енцефалопатія Acute hypertensive encephalopathy

Паренхіматозний крововилив Parenchymal hemorrhage

Ішемічний інсульт Ischemic stroke

Субарахноїдальний крововилив Subarachnoid hemorrhage

Транзиторна ішемічна атака Transient ischemic attack

146 / 200
Чоловік захворів гостро з підвищення температури до 40oC, головного болю у лобноскроневих ділян- ках, ломоти у всьому тілі. Непокоїв сухий кашель і біль за грудниною. Під час огляду дільничим лікарем виявлені склерит, гіперемія, ціаноз і зернистість слизової ротоглотки. Яке захворювання з числа ГРВ! у хворого? The man became acutely ill with an increase in temperature up to 40oC, headache in the frontotemporal areas, aches in the whole body. He was worried about a dry cough and pain behind the sternum. During the examination the district doctor detected scleritis, hyperemia, cyanosis, and granularity of the oropharyngeal mucosa. What disease from among the ENTs does the patient have?

РС-інфекція MS infection

Риновірусна інфекція Rhinovirus infection

Парагрип Paraflu

Аденовірусна інфекція Adenovirus infection

Грип Flu

147 / 200
В регіоні епідеміологічно небезпечному на сказ до травмпункту доставлено хлопчика 13ти років, якого щойно укусила лисиця. Черговий лікар провів первинну обробку рани. Які заходи невідкладної допомоги є обов’язковими? A 13-year-old boy who had just been bitten by a fox was brought to the trauma center in an epidemiologically dangerous region for rabies. The doctor on duty performed the initial treatment of the wound. What emergency measures are mandatory ?

Через 10 діб спостереження за необхідності призначити курс антира-бічних щеплень After 10 days of observation, it is necessary to appoint a course of anti-rabies vaccinations

Пасивна імунізація специфічним імуноглобуліном - інфільтрація в рану та в/м введення залишку і початок курсу антирабічних щеплень Passive immunization with specific immunoglobulin - infiltration into the wound and intravenous administration of the residue and the beginning of the course of anti-rabies vaccinations

Спостереження за потерпілим впродовж 10 днів і проведення курсу антирабічних щеплень Observation of the victim for 10 days and conducting a course of anti-rabies vaccinations

Проведення пасивної імунізації проти сказу та спостереження за потерпілим впродовж 7 днів Passive immunization against rabies and observation of the victim for 7 days

Негайно розпочати курс антирабі-чних щеплень - безумовні показання Immediately start a course of anti-rabies vaccinations - unconditional indications

148 / 200
Дівчинка 14-ти років внаслідок профузної маткової кровотечі знаходиться у важкому стані. З анамнезу: з 11 років періодичні шкірні геморагії, нерясні носові кровотечі. Об’єктивно: шкірні покриви і слизові бліді, визначається поліморфна геморагічна висипка. Пульс частий, ниткоподібний. ЧСС- 130/хв. Тони серця приглушені. ЧД- 30/хв., АТ- 70/40 мм рт.ст. Яка невідкладна тактика по відношенню до дитини? A 14-year-old girl is in serious condition due to profuse uterine bleeding. From the anamnesis: from the age of 11, periodic skin hemorrhages, profuse nosebleeds. Objectively: skin the integuments and mucous membranes are pale, a polymorphic hemorrhagic rash is determined. The pulse is frequent, filamentous. Heart rate - 130/min. Heart sounds are muffled. BH - 30/min., BP - 70/40 mmHg. What is the immediate tactic for the child?

Госпіталізація в хірургічне відділення Hospitalization in the surgical department

Госпіталізація у соматичне відділення Hospitalization in the somatic department

Лікування у домашніх умовах Treatment at home

Госпіталізація у реанімаційне відділення Hospitalization in intensive care unit

Лікування в амбулаторних умовах Outpatient treatment

149 / 200
Після автомобільної катастрофи у дівчинки 13-ти років має місце перелом кісток тазу. Вимушене положення на спині. Під час пальпації живота відзначається різка болісність у надлобковій ділянці. Відсутність самостійного сечовипускання з болісними покликами до нього. Чим зумовлений стан дитини? After a car accident, a 13-year-old girl has a pelvic fracture. She is forced to lie on her back. During palpation of the abdomen, sharp pain in the suprapubic area is noted. Absence of independent urination with painful calls to him. What is the cause of the child's condition?

Травма селезінки Spleen injury

Травма сечового міхура Bladder injury

Розрив кишечнику Intestinal rupture

Забій черевної стінки Abdominal wall bruise

Травма нирки Kidney injury

150 / 200
До амбулаторії доставлена дівчина 18-ти років через декілька годин після нападу бджіл. Скарги на набряк шиї у ділянці укусів, появу висипки на шкірі, що свербить. Об’єктивно: у ділянці лівого передпліччя набряк і гіперемія, на решті ділянок шкіри яскравий рожевий висип, місцями зливний до 5-6 см у діаметрі, сліди розчухувань. З боку внутрішніх органів - без особливостей, ЧСС- 104/хв., АТ-140/90 мм рт.ст. Яка загальна реакція розвинулася? An 18-year-old girl was brought to the outpatient clinic a few hours after a bee attack. Complaints of swelling of the neck in the area of ​​bites, the appearance of an itchy skin rash. About' objectively: swelling and hyperemia in the area of ​​the left forearm, on the rest of the skin a bright pink rash, in places confluent up to 5-6 cm in diameter, traces of scratching. From the side of the internal organs - no peculiarities, heart rate - 104/min., blood pressure - 140/ 90 mm Hg. What general reaction developed?

Початок розвитку анафілактичного шоку Start of development of anaphylactic shock

Загальний стан за типом кропив’янки General condition by type of urticaria

Набряк Квінке Quincke edema

Алергічний дерматит Allergic dermatitis

Гіперсенситивний васкуліт Hypersensitivity vasculitis

151 / 200
Хворий знаходився у судинному відділенні з приводу атеросклеротичної оклюзії стегнової артерії. Протягом двох годин непокоїли болі пекучого характеру за грудниною. Раптово знепритомнів. Пульс на сонних артеріях відсутній, дихання відсутнє. Які першочергові заходи реаніматологів? The patient was in the vascular department due to atherosclerotic occlusion of the femoral artery. Burning pains behind the sternum were bothering him for two hours. He suddenly fainted. There is no pulse on the carotid arteries, no breathing . What are the primary actions of resuscitators?

Перевести у відділення невідкладної кардіології Transfer to the emergency cardiology department

Інфузія нітрогліцерину Infusion of nitroglycerin

Внутрішньовенне введення атропіну Intravenous administration of atropine

Перевести у діагностичне відділення Transfer to diagnostic department

Почати непрямий масаж серця, штучну вентиляцію легень Start indirect heart massage, artificial lung ventilation

152 / 200
Хворий 72-х років знаходився у хірургічному відділенні з приводу діабетичної гангрени правої стопи. Ра- птово знепритомнів. Пульс на сонних артеріях не визначається, дихання поверхневе. Лікуючий лікар розпочав непрямий масаж серця. Як мають бути розташовані долоні лікаря? A 72-year-old patient was in the surgical department due to diabetic gangrene of the right foot. He suddenly fainted. The pulse on the carotid arteries is not determined, breathing is shallow. The attending physician started an indirect heart massage. How should the doctor's palms be positioned?

На верхній третині груднини On the upper third of the sternum

На межі верхньої і середньої третини груднини On the border of the upper and middle third of the sternum

На мєжі середньої і нижньої третини груднини On the border of the middle and lower third of the sternum

На мечоподібному відростку On the xiphoid process

На рукоятці груднини On the handle of the sternum

153 / 200
У жінки 46-ти років при проведенні пункції підключичної вени зліва виникла задишка, відставання в акті дихання лівої половини грудної клітки. Запідозрено розвиток пневмотораксу. Який метод дослідження необхідний для підтвердження діагнозу? A 46-year-old woman developed shortness of breath during a puncture of the left subclavian vein, a lag in the act of breathing in the left half of the chest. The development of pneumothorax is suspected. What research method is necessary for confirmation of the diagnosis?

Рентген ОГП X-ray OGP

КТ грудної порожнини CT chest cavity

УЗД Ultrasound

Біохімічні показники крові Biochemical indicators of blood

МРТ грудної порожнини MRI chest cavity

154 / 200
У потерпілого на радіаційно небезпечному об’єкті на 6-ту добу після опромінювання спостерігаються кровоточивість з ясен, помірна ало-пеція, пневмонія, тахікардія 98/хв., АТ- 100/60 мм рт.ст. У крові: лейк.-1,1 • 109/л, тромб.- 45 • 109/л, ШОЕ- 35 мм/год. Визначить ступінь тяжкості променевої хвороби та приблизну дозу опромінювання: On the 6th day after exposure, the victim at a radiation-hazardous facility has bleeding gums, moderate alopecia, pneumonia, tachycardia 98/min., Blood pressure - 100/60 mm Hg In the blood: leuk.-1.1 • 109/l, thrombo.- 45 • 109/l, ESR - 35 mm/h Determines the severity of radiation sickness and the approximate radiation dose:

іі (2-4 Гр) ii (2-4 Gy)

іV (понад 6 Гр) iV (over 6 Gy)

ііі (4-6 Гр) iii (4-6 Gy)

і (1-2 Гр) and (1-2 Gy)

Клінічна картина не зумовлена гострою променевою хворобою The clinical picture is not caused by acute radiation sickness

155 / 200
Використання якої з наведених методик штучної вентиляції легенів забезпечує максимальний дихальний об’єм? Using which of the following methods of artificial lung ventilation provides the maximum respiratory volume?

Хольгера-Нільсена Holger-Nielsen

Модифікований метод Сільвестра Modified Sylvester Method

Сільвестра Sylvester

З рота в рот Word of mouth

Підняття нижніх кінцівок Elevation of the lower limbs

156 / 200
У породіллі народився живий доношений хлопчик масою 4350 гр. з оцінкою за шкалою Апгар 3 бали. За- дні навколоплідні води меконіальні. При санації дихальних шляхів знайдена аспірація меконіальними водами. У новонародженого ЧСС- 80/хв. З чого повинні бути розпочаті реанімаційні заходи? A live, full-term boy weighing 4,350 g was born to a woman in labor with an Apgar score of 3. The amniotic fluid is meconial. Meconial fluid aspiration was found during airway sanitation A newborn has a heart rate of 80/min. From what should resuscitation measures be initiated?

Інгаляція кисню через маску Oxygen inhalation through a mask

Штучне дихання 'рот до рота' Mouth to mouth artificial respiration

Стимуляція дихання поплескуванням по сідницях, напрям в ніс струменя кисню, окроплення холодною водою Stimulation of breathing by patting the buttocks, the direction of the oxygen stream into the nose, sprinkling with cold water

Введення у вену етімізола, глюкози, кокарбоксилази, глюконата кальцію Introduction of etimisol, glucose, cocarboxylase, calcium gluconate into a vein

Санація бронхіального дерева під контролем ларингоскопа, інтубація, ШВЛ Rehabilitation of the bronchial tree under the control of a laryngoscope, intubation, mechanical ventilation

157 / 200
У хворого, що знаходиться в реанімаційному відділенні інфекційної клініки, виникла фібриляція шлуночків, зареєстрована на кардіомоніторі. Реанімаційні заходи слід розпочати з: The patient, who is in the intensive care unit of the infectious disease clinic, developed ventricular fibrillation, registered on the cardiomonitor. Resuscitation measures should be started from:

Електричної дефібриляції Electric defibrillation

Прекардіального удару Precardiac attack

Введення етацизіну Introduction of etacizin

Введення лідокаїну Lidocaine administration

Введення атропіну Atropine administration

158 / 200
Водій 37-ми років попав в автомобільну катастрофу та отримав закриту травму живота без ушкодження передньої черевної стінки. З’явився сильний біль в животі, напруження м’язів живота, розвиваються симптоми шоку. Якої тактики слід дотримуватись в цій ситуації? A 37-year-old driver was involved in a car accident and received a closed abdominal injury without damage to the anterior abdominal wall. He developed severe abdominal pain, abdominal muscle tension , symptoms of shock are developing. What tactics should be followed in this situation?

Ввести наркотичні аналгетики Enter narcotic analgesics

Негайно транспортувати в хірургічне відділення. Протишокові заходи здійснювати під час транспортування Immediately transport to the surgical department. Perform anti-shock measures during transport

Провести інфузію реополіглюкіну на місці пригоди Infuse rheopolyglucin at the scene of the adventure

На місці пригоди ввести серцеві глікозиди Inject cardiac glycosides at the scene of the accident

На місці пригоди почати оксигенотерапію Start oxygen therapy at the scene of the adventure

159 / 200
У сільській місцевості, в полі, у жінки 52-х років раптово погіршився стан: впала на землю, на питання не відповідає, очі не відкриває, дихання немає, шкірні покриви обличчя та рук бліді. Хто має розпочати надання медичної допомоги в такій ситуації? In the countryside, in the field, a 52-year-old woman's condition suddenly worsened: she fell to the ground, does not answer questions, does not open her eyes, is not breathing, the skin of the face and hands is pale. Who should initiate medical assistance in such a situation?

Спеціалізована бригада швидкої медичної допомоги Specialized ambulance team

Дільничий лікар District doctor

Співробітник, що пройшов спеціальну підготовку Specially trained employee

Районний терапевт District therapist

Свідок події Event witness

160 / 200
Зі слів матері у дворічної дитини на фоні повного соматичного благополуччя виникла профузна кровотеча алою кров’ю з домішками темних згустків без больового синдрому та при відсутності подібних симптомів у анамнезі. Яке захворювання найбільш імовірне у дитини? According to the mother, a two-year-old child, against the background of complete somatic well-being, had profuse bleeding with scarlet blood mixed with dark clots without pain syndrome and in the absence of similar symptoms in the anamnesis. What disease is the most likely for the child?

Тріщина заднього проходу Fissure of anus

Гострий апендицит Acute appendicitis

Дивертикул Меккеля Meckel's diverticulum

Виразковий коліт Ulcerative colitis

Інвагінація здухвинної кишки Intussusception of the ileum

161 / 200
До лікаря сільської амбулаторії звернувся хлопчик 16-ти років зі скаргами на підвищення to тіла до 38,5oC та виразковий дефект шкіри на передпліччі. Об’єктивно: виразка безболісна з ущільненими набряклими краями. З анамнезу: хлопчик допомагає батькам обробляти шкіру тварин. Лікар запідозрив сибірку. Який термін спостереження за контактними особами? A 16-year-old boy turned to the doctor of a rural outpatient clinic with complaints of an increase in body temperature to 38.5oC and an ulcerated skin defect on the forearm. Objectively: the ulcer is painless with thickened, swollen edges. From the anamnesis: the boy helps his parents process animal skins. The doctor suspected anthrax. What is the period of observation of contact persons?

2 доби 2 days

8-9 діб 8-9 days

21 доба 21 days

14 діб 14 days

Спостереження не проводиться No monitoring

162 / 200
У дівчинки 3-х тижнів тривале блювання, пронос, м’язова гіпотонія, вагу не набирає. Клітор значно збільшений. Калій крові - 6,5 ммоль/л, натрій - 125 ммоль/л. Невідкладні лікувальні заходи насамперед передбачають: A 3-week-old girl has prolonged vomiting, diarrhea, muscle hypotonia, does not gain weight. The clitoris is significantly enlarged. Blood potassium - 6.5 mmol/l , sodium - 125 mmol/l. Urgent medical measures primarily include:

Парентеральне харчування Parenteral nutrition

Антибіотикотерапія Antibiotic therapy

Гемодіаліз Hemodialysis

Оральна регідратація Oral rehydration

Замісна терапія глюко- та мінера-локортикоїдами Substitution therapy with gluco- and mineralocorticoids

163 / 200
У пацієнта під час протезування кульшового суглоба, що проводилося під загальною анестезією, відбулася зупинка кровообігу. У даній ситуації: The patient had a circulatory arrest during the hip replacement performed under general anesthesia. In this situation:

- -

Необхідно проведення непрямого масажу серця Indirect cardiac massage is required

Масаж серця не проводиться, достатньо проведення штучної вентиляції легень Heart massage is not performed, artificial lung ventilation is sufficient

Необхідна лапаротомія і проведення масажу серця через діафрагму Laparotomy and cardiac massage through the diaphragm is necessary

Необхідна торакотомія і проведення прямого масажу серця Thoracotomy and direct heart massage is necessary

164 / 200
До приймального відділення надійшов пацієнт непритомний, реакція на больовий подразник відсутня, дихання самостійне. АТ- 90/50 мм рт.ст. Даний стан можна розцінити як: The patient arrived at the reception department unconscious, there is no reaction to a painful stimulus, breathing is independent. Blood pressure - 90/50 mm Hg. This condition can be considered as:

Ступор Stupor

Сопор Sopor

Поверхневе оглушення Surface Stun

Кома Comma

Помірне оглушення Moderate Stun

165 / 200
Після ліквідації пожежі у задимленому приміщенні лікарі швидкої допомоги знайшли двох людей з опіками кінцівок. Об’єктивно: запаморочення, головний біль, зіниці помірно розширені, шкіра і слизові оболонки рожеві, тони серця глухі, тахікардія, помірна гіпертензія. Яку допомогу треба надати насамперед? After extinguishing the fire in the smoky room, emergency doctors found two people with burns on their limbs. Objectively: dizziness, headache, pupils moderately dilated, skin and mucous membranes pink, dull heart sounds, tachycardia, moderate hypertension. What help should be given first?

Ввести знеболювальні засоби Enter painkillers

Ввести антидоти Enter antidotes

Накласти асептичні пов’язки на рани Put aseptic dressings on wounds

Винести на свіже повітря і розпочати оксигенацію Take to fresh air and start oxygenation

Іммобілізувати ушкоджені кінцівки підручними засобами Immobilize damaged limbs with improvised means

166 / 200
Дитина 3-х років через 3 дні після забою правої ноги скаржиться на біль в ураженій кінцівці, який посилюється при активних рухах. При огляді виявлено підвищення місцевої температури над правим стегном, його набряк, порушення функції стегнового суглоба, біль при перкусії метафізу. В яке відділення необхідно госпіталізувати дитину для надання допомоги? Three days after the slaughter of the right leg, a 3-year-old child complains of pain in the affected limb, which worsens with active movements. During the examination, an increase in local temperature over the right thigh, its swelling, hip joint dysfunction, pain when percussing the metaphysis. In which department should the child be hospitalized for assistance?

Гематологічне Hematological

Педіатричне Pediatric

Хірургічне Surgical

Соматичне Somatic

Кардіоревматологічне Cardiorheumatic

167 / 200
У роділлі 28-ми років під час дру- гого періоду пологів виникла раптова зупинка серця та дихання. В анамнезі вроджена вада серця - дефект між-шлуночкової перегородки, серцева недостатність іі А ст. На 5-й хвилині серцево-легеневої реанімації, яка почалась вчасно, відновлення серцевої діяльності не зареєстровано. Який прогноз при подальшій реанімації відносно відновлення серцевої діяльності? In a 28-year-old woman in labor, during the second stage of labor, sudden cardiac and respiratory arrest occurred. She has a history of congenital heart disease - a defect of the interventricular septum, cardiac failure of II and A stage. At the 5th minute of cardiopulmonary resuscitation, which began on time, the recovery of cardiac activity was not registered. What is the prognosis for further resuscitation regarding the recovery of cardiac activity?

Повне одужання Full recovery

Можливе відновлення синусового ритму Sinus rhythm recovery is possible

Повністю відновити ритм неможливо It is impossible to completely restore the rhythm

Повне одужання з наступним погіршанням Full recovery followed by deterioration

Відновлення ритмічних скорочень з окремими екстрасистолами Restoration of rhythmic contractions with separate extrasystoles

168 / 200
Бригада швидкої медичної допомоги прибула за викликом на автоза-правочну станцію, де відбулась аварія і вилилась велика кількість низькоо-ктанового бензину. Працівники АЗС скаржаться на запаморочення, збудження, нудоту, блювання. В чому полягає тактика лікаря? The emergency medical team arrived on call to a gas station where an accident occurred and a large amount of low-octane gasoline was spilled. The gas station employees complain of dizziness, excitement, nausea, vomiting. What are the doctor's tactics?

Надати інгаляцію кисню Provide oxygen inhalation

Вивести людей із зони, насиченої парами бензину Remove people from the area saturated with gasoline vapors

Призначити атропін Prescribe atropine

Призначити кордіамін Prescribe cordiamine

Промити шлунок постраждалим Wash the victim's stomach

169 / 200
У новородженого згідно шкали Апгар виявлено 4 бали: субтотальний ціаноз, поодинокі дихальні рухи, м’язова гіпотонія, слабкий крик, ЧСС-60/хв, арефлексія. Пологи своєчасні, маса тіла 3200 г, спостерігалось обвит-тя пуповини навколо шиї. У цій ситуації необхідно: According to the Apgar scale, the newborn had 4 points: subtotal cyanosis, single respiratory movements, muscle hypotonia, weak cry, HR-60/min, areflexia. Childbirth timely, body weight 3200 g, umbilical cord wrapping around the neck was observed. In this situation, it is necessary:

Невідкладна дефібриляція Urgent defibrillation

Проведення реанімаційних заходів Carrying out resuscitation measures

Рефлекторна стимуляція дихання Reflex stimulation of breathing

Інтенсивне спостереження Intensive monitoring

Інгаляція кисню через носові катетери Oxygen inhalation through nasal catheters

170 / 200
У породіллі 25-ти років через 5 хвилин після народження дитячого місця з’явилися озноб, задишка, тахікардія. АТ- 80/40 мм рт.ст., to- 37,8oC. Який найбільш імовірний діагноз? A 25-year-old woman in labor developed chills, shortness of breath, and tachycardia 5 minutes after giving birth. Blood pressure - 80/40 mm Hg, to- 37.8oC. What is the most likely diagnosis?

Гострий апендицит Acute appendicitis

Набряк легень Pulmonary edema

Розрив матки Rupture of uterus

Респіраторна вірусна інфекція Respiratory viral infection

Емболія навколоплідними водами Amniotic fluid embolism

171 / 200
Дитина 9-ти років впала з висоти 1,5 м на металеву трубу ділянкою промежини. При надходженні скаржиться на біль та відчуття розпирання в промежині при спробі сечовипускання, виділення крові з уретри. Об’єктивно: в ділянці промежини - значна гематома та інфільтрація м’яких тканин, дно сечового міхура пальпується на рівні пупка, сечовипускання відсутнє, з уретри виділяється свіжа кров. Який найбільш імовірний діагноз? A 9-year-old child fell from a height of 1.5 m onto a metal pipe through the perineum. Upon admission, he complains of pain and a feeling of distention in the perineum when trying to urinate, discharge of blood from the urethra. Objectively: in the perineum there is a significant hematoma and infiltration of soft tissues, the bottom of the bladder is palpable at the level of the navel, there is no urination, fresh blood is released from the urethra. What is the most likely diagnosis?

Перелом кісток тазу Fracture of pelvic bones

Травматична гематома промежини Traumatic perineal hematoma

Травматичний розрив нирки Traumatic rupture of the kidney

Розрив уретри Rupture of the urethra

Розрив сечового міхура Bladder rupture

172 / 200
У хворої 65-ти років, яка хворіє на гіпертонічну хворобу ііі ст., раптово погіршився стан: артеріальний тиск підвищився до 220/140 мм рт.ст., з’явилися задишка, кашель з виділенням кров’янистого харкотиння, клекотіння у грудях. До якої категорії відноситься зазначений стан з точки зору терміновості зниження артеріального тиску? A 65-year-old patient suffering from hypertension of the 3rd grade suddenly worsened: blood pressure rose to 220/140 mm Hg. shortness of breath, coughing with bloody sputum, rattling in the chest appeared. To which category does the specified condition belong in terms of the urgency of lowering blood pressure?

Артеріальний тиск необхідно знизити упродовж однієї доби Blood pressure should be reduced within one day

Артеріальний тиск необхідно знизити упродовж 2-х діб Blood pressure should be lowered within 2 days

Артеріальний тиск необхідно знизити упродовж однієї години Blood pressure must be reduced within one hour

Артеріальний тиск необхідно знизити упродовж 3-6 годин Blood pressure should be lowered within 3-6 hours

Артеріальний тиск знижувати не слід Blood pressure should not be lowered

173 / 200
В лабораторії де проводились досліди з радіоактивним йодом, відбулася аварія, внаслідок якої виникло радіоактивне забруднення приміщення та прилеглої території. Які препарати призначають постраждалим з метою профілактики променевої хвороби? In the laboratory where experiments with radioactive iodine were conducted, an accident occurred, as a result of which radioactive contamination of the room and the surrounding area occurred. What drugs are prescribed to the victims to prevent radiation sickness?

Йодид калію Potassium iodide

Знє6олюючі Destroying

Антибіотики Antibiotics

Інфузійна терапія Infusion therapy

Антациди Antacids

174 / 200
До медичного пункту полку з місця літніх польових навчань з гарматної стрілянини доставлено екіпаж танку з однотиповими скаргами та східною клінічною картиною: тяжкий стискаючий головний біль у скронях, нудота, запаморочення; у одного з них блювання с пригніченням свідомості, дезорієнтація. Об’єктивно: обличчя дещо пастозне, склери ін’єковані. Артеріальний тиск підвищений, тахікардія. У одного - порушення дихання. Яка найбільш імовірна патологія у військовослужбовців? A tank crew was brought to the regiment's medical center from the place of summer field artillery exercises with the same complaints and an oriental clinical picture: severe squeezing headache in the temples, nausea, dizziness ; one of them has vomiting with suppression of consciousness, disorientation. Objectively: the face is somewhat pasty, the sclera is injected. Blood pressure is elevated, tachycardia. One has breathing disorders. What is the most likely pathology in military personnel?

Отруєння пороховими газами Poisoning by powder gases

Мінно-вибухова травма, контузія Mine-explosive injury, contusion

Гостре шлунково-кишкове захворювання Acute gastrointestinal disease

Отруєння сурогатами алкоголю Alcohol surrogate poisoning

Перегрівання Overheating

175 / 200
Дитина 5-ти років скаржиться на лихоманку, пронос, блювання та запаморочення. У крові: Ht- 0,55, Nа+-118 ммоль/л, К+- 5,9 ммоль/л. Які розлади водно-електролітного балансу спостерігаються? A 5-year-old child complains of fever, diarrhea, vomiting and dizziness. In the blood: Ht- 0.55, Na+-118 mmol/l, K +- 5.9 mmol/l. What disorders of the water-electrolyte balance are observed?

Гіпотонічна дегідратація, гіпокаліє-мія Hypotonic dehydration, hypokalemia

Ізотонічна дегідратація, гіперкаліє-мія Isotonic dehydration, hyperkalemia

Гіперкаліємія Hyperkalemia

Розладів водно-електролітного балансу немає There are no water-electrolyte balance disorders

Гіпотонічна дегідратація, гіперкалі-ємія Hypotonic dehydration, hyperkalemia

176 / 200
6- місячна дитина госпіталізована з приводу 3-х разового блювання, рідких випорожнень більше 10 разів на добу. Об’єктивно: неспокійна, риси обличчя загострені, шкіра та слизові оболонки бліді та сухі, велике тім’ячко запале, to- 38,7oC, ЧСС- 162/хв., тони серця глухі, живіт здутий, діурез зменшений, дефіцит маси тіла - 10%. У крові: Ht- 50%, К+- 3,4 ммоль/л, Nа+- 154 ммоль/л. Який провідний клінічний синдром? A 6-month-old child was hospitalized due to vomiting 3 times, loose stools more than 10 times a day. Objectively: restless, facial features sharpened, skin and mucous membranes are pale and dry, the big head is inflamed, to- 38.7oC, heart rate- 162/min., heart sounds are dull, the stomach is distended, diuresis is reduced, body weight deficit - 10%. In the blood: Ht- 50%, K+- 3.4 mmol/l, Na+- 154 mmol/l. What is the leading clinical syndrome?

Гіпертермічний синдром Hyperthermic syndrome

Синдром ентероколіту Enterocolitis syndrome

Соледефіцитний ексикоз Sole deficiency exicosis

Вододефіцитний ексикоз Water deficit exicosis

Ацетонемічний синдром Acetonemic syndrome

177 / 200
Роділля у терміні 38 тижнів вагітності звернулась у лікарню зі скаргами на регулярні переймоподібні болі внизу живота протягом 2 годин. На обліку не перебувала. 2 тижні тому хворіла грипом з високою температурою тіла. Протягом 2-х діб рухів плоду не відчуває. Серцебиття плоду не вислуховується. Який найбільш інформативний метод діагностики стану плоду? A woman in labor at 38 weeks of pregnancy went to the hospital complaining of regular spasm-like pains in the lower abdomen for 2 hours. She was not registered. 2 weeks ago she had the flu with high by body temperature. During 2 days, the fetus does not feel movements. The heartbeat of the fetus is not heard. What is the most informative method of diagnosing the state of the fetus?

Амніоскопія Amnioscopy

Ультразвукове дослідження Ultrasound examination

Рентгенографічне дослідження X-ray examination

Амніоцентез Amniocentesis

Кардіотокографія плоду Cardiotocography of the fetus

178 / 200
У вагітної з пізнім гестозом на тлі АТ- 180/110 мм рт.ст. виникли скарги на сильний головний біль, мерехтіння 'мушок'перед очима. Потім потемніло в очах, розвинувся напад судом. Які першочергові дії? A pregnant woman with late gestosis against the background of blood pressure of 180/110 mm Hg complained of a severe headache, flickering of 'flies' before her eyes. Then it got dark in the eyes, a seizure has developed. What are the first actions?

Госпіталізація вагітної до відділення РІТ Hospitalization of a pregnant woman to the RIT department

Непрямий масаж серця Indirect heart massage

Забезпечення прохідності дихальних шляхів Ensuring airway patency

Прямий масаж серця Direct heart massage

ШВЛ VENTILATOR

179 / 200
Моряка після аварії витягнули з холодної води, де він перебував впродовж 20-30 хвилин. Шкіра бліда з мармуровим малюнком, акроціаноз, м’язи ригідні, свідомість пригнічена. ЧД-9/хв., ЧСС- 45/хв., АТ- 160/90 мм рт.ст. Ректальна температура 33oC. Який найбільш імовірний діагноз? After the accident, the sailor was pulled out of the cold water, where he stayed for 20-30 minutes. The skin is pale with a marble pattern, acrocyanosis, muscles are rigid, consciousness is depressed. BH-9/min., HR-45/min., BP- 160/90 mm Hg. Rectal temperature 33oC. What is the most likely diagnosis?

Загальне переохолодження легкого ступеня General mild hypothermia

Загальне переохолодження тяжкого ступеня General severe hypothermia

Інфаркт міокарда Myocardial infarction

Утоплення Drowning

Загальне переохолодження середнього ступеня General hypothermia of medium degree

180 / 200
У дівчинки 6-ти років скарги на біль у ділянці піхви та кров’янисті виділення, загальне нездужання, пла- ксивість, замкненість після прогулянки з падлюками. Під час огляду на внутрішній поверхні стегон синці та садна. Яка найбільш імовірна причина травми? A 6-year-old girl complains of pain in the vagina and bleeding, general malaise, tearfulness, and withdrawal after a walk with peddlers. During examination of the inner surface of the thighs bruises and sores. What is the most likely cause of the injury?

Гостре запалення додатків матки Acute inflammation of the uterine appendages

Травма зовнішніх статевих органів Injury of external genitalia

Ізольоване менархе Isolated menarche

Вульвовагініт Vulvovaginitis

Зґвалтування Rape

181 / 200
У постраждалого після падіння з висоти на спину лікар швидкої допомоги запідозрив перелом лівої лопатки. Після проведення знеболювання проведена транспортна іммобілізація. Чим оптимально проводити транспортну іммобілізацію при подібних пошкодженнях? After falling from a height onto his back, the emergency doctor suspected a fracture of the left scapula. After anesthesia, transport immobilization was carried out. What is the best way to carry out transport immobilization for such injuries?

Гіпсова лонгета Gypsum slab

Шина ЦІТО Tire THIS

Шина Крамера Kramer Tire

Підручні засоби Helps

Пов’язка Дезо Deso Bandage

182 / 200
В результаті ДТП пацієнт отримав травму ліктьового суглоба. До місця події прибула бригада швидкої допомоги. За допомогою яких шин необхідно провести транспортну іммобілізація даного суглоба. As a result of a road accident, the patient suffered an injury to the elbow joint. An ambulance team arrived at the scene. Which tires should be used for transport immobilization of this joint.

Шина Дітеріхса Tire of Dieterichs

Шина Крамера Kramer Tire

Гіпсова шина до верхньої третини плеча Gap cast to the upper third of the shoulder

Шина Еланського Shyna Elansky

Гіпсова шина до здорової лопатки Plaster cast to a healthy scapula

183 / 200
Пацієнт доставлений до приймального відділення з судомами, відсутністю свідомості, анізокорією, брадикардією до 40/хв. З анамнезу відомо, що пацієнт декілька годин тому сильно вдарився головою, знепритомнів, однак потім прийшов до тями, але протягом декількох годин його стан поступово погіршувався. Який метод дослідження буде найбільш інформативним для постановки остаточного діагнозу? The patient was brought to the reception department with convulsions, lack of consciousness, anisocoria, bradycardia up to 40/min. It is known from the anamnesis that the patient hit his head a few hours ago, fainted , but then he came to his senses, but over the course of several hours his condition gradually worsened. What research method will be the most informative for making a final diagnosis?

Комп’ютерна томографія головного мозку Computed tomography of the brain

Рентген кісток черепа X-ray of skull bones

Ехоенцефалоскопія Echoencephaloscopy

Реоенцефалографія Rheoencephalography

Спінальна пункція Spinal puncture

184 / 200
На території селища, де живуть 200 мешканців, встановлено забруднення ґрунту ізотопами стронцію-89 та стронцію-90. В якій тканині найбільш імовірно буде виявлено накопичення даного ізотопу? On the territory of the village, where 200 residents live, contamination of the soil with strontium-89 and strontium-90 isotopes has been established. In which tissue is the accumulation of this isotope most likely to be detected?

Нервова тканина Nervous tissue

Серце та судини Heart and vessels

М’язова тканина Muscle tissue

Деривати шкіри Derivate skins

Кісткова тканина Bone tissue

185 / 200
У постраждалого на тлі гострого отруєння фосгеном розвинулася клініка дихальної та серцево-судинної недостатності. Який показник є несприятливим симптомом щодо прогнозу перебігу отруєння? A victim of acute phosgene poisoning developed a clinic of respiratory and cardiovascular insufficiency. Which indicator is an unfavorable symptom regarding the prognosis of the course of poisoning?

Підвищення температури тіла до 37,5oC Increasing body temperature to 37.5oC

Загальна слабкість General weakness

Згущення крові Blood clotting

Тахікардія до 120/хв. Tachycardia up to 120/min.

Зниження артеріального тиску нижче 100 мм рт.ст. Blood pressure drop below 100 mmHg

186 / 200
У хворої 45-ти років перелом правого стегна. На догоспітальному етапі для транспортної іммобілізації застосовано шину Дітеріхса. З якою метою застосовується іммобілізація в даній шині? A 45-year-old patient has a right hip fracture. At the pre-hospital stage, a Dieterichs splint was used for transport immobilization. What is the purpose of immobilization in this splint?

Для запобігання м’язовій контрактурі в зоні перелому To prevent muscle contracture in the fracture area

Для ліпшої фіксації пов’язки For better fixation of the bandage

Шина Дітеріхса для нижньої кінцівки не застосовується Dieterichs splint for the lower extremity is not used

Для репозиції відламаних сегментів кістки For repositioning broken bone segments

Як атавізм Like atavism

187 / 200
Вагітна в терміні 27-28 тижнів звернулась зі скаргами до лікаря жіночої консультації зі скаргами на постійний ниючий біль внизу живота та відсутність відчуття рухів плоду. Які обстеження необхідно провести першочергово? A pregnant woman at 27-28 weeks complained to the doctor of the women's consultation with complaints of constant aching pain in the lower abdomen and lack of feeling of fetal movements. What examinations should be carried out as a priority ?

Визначення рівня плацентарного лактогену Determining the level of placental lactogen

Вагінальний огляд вагітної Vaginal examination of a pregnant woman

Визначення рівня хоріонічного гонадотропіну Determining the level of chorionic gonadotropin

Ультразвукове обстеження плоду Ultrasound examination of the fetus

Кардіотокографія плоду Cardiotocography of the fetus

188 / 200
Чоловік 85-ти років під час сміху закашлявся, після чого розвинувся ціаноз обличчя і кінцівок, дихання утруднене, при огляді ротової порожнини - відсутня частина зубного протезу. Стан хворого погіршувався, відмічалися тривалі напади виснажливого кашлю, біль за грудниною, блювання. Яка повинна бути тактика лікаря швидкої допомоги? An 85-year-old man coughed while laughing, after which cyanosis of the face and limbs developed, breathing became difficult, during an examination of the oral cavity, a part of the denture was missing. The patient's condition worsened, prolonged bouts of debilitating cough, chest pain, vomiting were noted. What should be the tactics of an emergency physician?

Термінова госпіталізація в кардіологічне відділення Urgent hospitalization in the cardiology department

Термінова госпіталізація в хірургічне відділення Urgent hospitalization in the surgical department

Термінова трахеотомія Urgent tracheotomy

Термінова госпіталізація в ЛОР-відділення Urgent hospitalization in the ENT department

Термінова госпіталізація у відділення реанімації Urgent hospitalization in the intensive care unit

189 / 200
У хлопчика 2-х років з вродженою вадою серця - тетрадою Фалло, виник задишковоціанотичний криз. Якою повинна бути тактика лікаря в даній ситуації? A 2-year-old boy with a congenital heart defect - tetrad of Fallot developed a dyspnoea-cyanotic crisis. What should be the doctor's tactics in this situation?

Ввести строфантин та госпіталізувати у відділення реанімації та інтенсивної терапії Inject strophanthin and hospitalize in intensive care unit

Госпіталізувати у відділення реанімації та інтенсивної терапії Hospitalize in intensive care unit

Ввести еуфілін та госпіталізувати у відділення реанімації та інтенсивної терапії Inject euphilin and hospitalize in intensive care unit

Ввести пропранолол та госпіталізувати у відділення реанімації та інтенсивної терапії Inject propranolol and hospitalize in intensive care unit

Ввести глюкокортикоїди та госпіталізувати у відділення реанімації та інтенсивної терапії Inject glucocorticoids and hospitalize in intensive care unit

190 / 200
При аварії на ядерному реакторі відбувся викид в навколишнє середовище значної кількості радіоізотопів йоду. Який шлях надходження їх в організм має найбільш важливе значення в ранній період після аварії? In the event of a nuclear reactor accident, a significant amount of radioisotopes of iodine was released into the environment. What route of their entry into the body is of the most importance in the early period after the accident?

Через рани та опікові поверхні Due to wounds and burn surfaces

Через слизові Through mucous membranes

Аліментарний Alimony

Інгаляційний Inhalation

Через шкіру Through the skin

191 / 200
До пологового відділення надійшла роділля зі скаргами на різкий головний біль, нудоту. АТ-180/110 мм рт.ст. По катетеру виділилось 20 мл концентрованої сечі, білок сечі - 2,9 г/л. Яке першочергове призначення лікаря? A woman in labor came to the maternity ward with complaints of a sharp headache, nausea. BP-180/110 mm Hg. 20 ml of concentrated urine, protein urine - 2.9 g/l. What is the primary appointment of the doctor?

Болюсне внутрішньовенне введення 25% розчину магнезії Bolus intravenous administration of 25% magnesium solution

Клофелін в/м Clofelin IV

Ніфедипін під язик Sublingual nifedipine

25% розчин магнезії 10 мл в/м 25% magnesium solution 10 ml in/m

Допегіт під язик Dopegit under the tongue

192 / 200
У роділлі 29-ти років із вродженою вадою серця в І періоді пологів з’явилися кашель з харкотинням, задишка, болі за грудниною, тахікардія. АТ- 90/60 мм рт.ст., to- 37,8oC. Роділля займає вимушене сидяче положення. Який найбільш імовірний діагноз? A 29-year-old woman in labor with a congenital heart defect developed cough with sputum, shortness of breath, chest pains, tachycardia in the first period of labor. AT-90/ 60 mm Hg, to- 37.8oC. The woman in labor takes a forced sitting position. What is the most likely diagnosis?

Набряк легень Pulmonary edema

Інфаркт міокарда Myocardial infarction

Розрив матки Rupture of uterus

Напад бронхіальної астми Bronchial asthma attack

Емболія навколоплідними водами Amniotic fluid embolism

193 / 200
У дитини 10-ти років через 2 місяці після трансфузії донорської крові з’явились жовтяниця, гепатоспле-номегалія, фебрильна температура, слабкість. При лабораторному дослідженні: АлАТ- 40 (ммоль/чл), загальний білірубін - 80 мкмоль/л. У крові високий титр антитіл до M-аntі-CMV-іgM, ПЛР +++. Який найбільш імовірний діагноз? A 10-year-old child developed jaundice, hepatosplenomegaly, febrile temperature, and weakness 2 months after the transfusion of donor blood. During the laboratory examination: AlAT- 40 (mmol/l), total bilirubin - 80 μmol/l. High titer of antibodies to M-anti-CMV-иgM in the blood, PCR +++. What is the most likely diagnosis?

Дискінезія жовчних шляхів Biliary dyskinesia

Цитомегаловірусна інфекція Cytomegalovirus infection

Цироз печінки Liver cirrhosis

Гострий холецистит Acute cholecystitis

Вірусний гепатит В Viral hepatitis B

194 / 200
У 70-літньої жінки через 48 годин після оперативного лікування хвороби кишечнику з’явилися скарги на нестачу повітря, задишку, кашель та біль у грудній клітці. Діагноз легеневої емболії підтверджений, проводиться адекватна терапія. На тлі лікування і значного покращання стану протягом кількох діб наступила повторна емболія. Яка лікувальна тактика? A 70-year-old woman complained of shortness of breath, shortness of breath, cough, and chest pain 48 hours after surgical treatment of intestinal disease. Diagnosis of pulmonary embolism confirmed, adequate therapy is carried out. Against the background of treatment and a significant improvement of the condition, a repeated embolism occurred within a few days. What are the treatment tactics?

Емболектомія з легеневої артерії Pulmonary artery embolectomy

Підвищення дози антикоагулянтів Increasing the dose of anticoagulants

Постановка кава-фільтру Setting coffee filter

Перев’язка аорти Ligation of the aorta

Продовження терапії Continuation of therapy

195 / 200
Хвора 64-х років з невралгією лицевого нерва після натужування в ту- алеті впала без свідомості. Черговим лікарем встановлена клінічна смерть і розпочаті реанімаційні заходи. Під час зовнішнього масажу серця виник перелом ребер. Які повинні бути дії реаніматора? A 64-year-old patient with neuralgia of the facial nerve fell unconscious after straining in the toilet. The doctor on duty established clinical death and started resuscitation measures. During external massage the heart suffered a rib fracture. What should be the actions of the resuscitator?

Припинити реанімаційні заходи Stop resuscitation measures

Перекласти хворого на м’яку поверхню Transfer the patient to a soft surface

Виконувати непрямий масаж з обмеженням глибини до 1-2 см Perform indirect massage with a depth limit of 1-2 cm

Продовжити реанімаційні заходи Continue resuscitation measures

Викликати реаніматолога Call an intensivist

196 / 200
Військовослужбовця доставлено на сортувальний майданчик медичної роти через 2 години після ядерного вибуху. Індивідуальний дозиметр відсутній. Скаржиться на загальну слабкість, нудоту, головний біль, спрагу. Безпосередньо після вибуху виникло багаторазове блювання, потім знепритомнів на 20 хв. При огляді загальмований, адинамічний, обличчя гіпе-ремоване, мова утруднена, часті спроби блювання. Ps- 120/хв., слабкого наповнення, тони серця приглушені. АТ-90/60 мм рт.ст. Дихання самостійне, везикулярне, ЧД- 18/хв. Який найбільш імовірний діагноз? The serviceman was brought to the medical company triage site 2 hours after the nuclear explosion. There is no individual dosimeter. He complains of general weakness, nausea, headache, thirst. Immediately after the explosion repeated vomiting occurred, then he fainted for 20 min. On examination, he was inhibited, adynamic, his face was hyper-repressed, speech was difficult, frequent attempts to vomit. Ps- 120/min., weak filling, heart sounds were muffled. BP-90/60 mm Hg. St. Breathing is independent, vesicular, BH-18/min. What is the most likely diagnosis?

Гостра променева хвороба, церебральна форма, іV ступінь важкості Acute radiation sickness, cerebral form, and V degree of severity

Гостра променева хвороба, прихований період Acute radiation sickness, latent period

Гостра променева хвороба іі (середнього) ступеня важкості Acute radiation sickness of the second (medium) degree of severity

Гостра променева реакція Acute radiation reaction

Гостра променева хвороба, кишкова форма, іV ступінь важкості Acute radiation sickness, intestinal form, and V degree of severity

197 / 200
Хворий 35-ти років, ветеринар, надійшов на стаціонарне лікування зі скаргами на підвищення температури до 39,5oC протягом останніх 2-х тижнів, підвищену пітливість, збільшення периферичних лімфовузлів, біль у крижовому відділі хребта. Лікуючий лікар запідозрив у хворого бруцельоз. Яким із методів досліджень можна підтвердити попередній діагноз? A 35-year-old patient, a veterinarian, came to inpatient treatment with complaints of an increase in temperature to 39.5oC during the last 2 weeks, increased sweating, an increase in peripheral lymph nodes, pain in the sacral region of the spine. The attending physician suspected the patient of brucellosis. What research methods can be used to confirm the previous diagnosis?

Реакція Райта Wright Reaction

Реакція Гоффа-Бауера Hoff-Bauer reaction

Реакція Відаля Vidal reaction

Реакція Пауля-Буннеля Paul-Bunnel reaction

Реакція Вейля Weyl reaction

198 / 200
Жінці 35-ти років з приводу пневмонії був призначений ампіцилін. Через 30 хвилин після внутрішньом’-язової ін’єкції препарату хвора відчула різку слабкість, свербіж шкіри обличчя та рук, нудоту; з’явилися кашель, задишка, біль у грудній клітці. Об’єктивно: ціаноз, набряк повік та обличчя з червоними висипаннями. Ps- 120/хв, АТ- 70/20 мм рт.ст. Тони серця глухі, дихання часте, поверхневе, з різнокаліберними сухими хрипами. Варикозне розширення вен на правій гомілці. Яка найбільш вірогідна причина раптового погіршення стану хворої? A 35-year-old woman was prescribed ampicillin for pneumonia. 30 minutes after the intramuscular injection of the drug, the patient felt sharp weakness, itching of the skin of the face and hands, nausea; cough, shortness of breath, chest pain appeared. Objectively: cyanosis, swelling of the eyelids and face with red rashes. Ps- 120/min, BP- 70/20 mm Hg. Heart sounds are dull, breathing is frequent, shallow, with dry wheezes of various calibers. Varicose veins on the right lower leg. What is the most likely cause of the sudden deterioration of the patient's condition?

Набряк Квінке Quincke edema

Анафілактичний шок Anaphylactic shock

Тромбоемболія гілок легеневої артерії Thromboembolism of the branches of the pulmonary artery

Кропивниця Hives

Астматичний напад Asthma attack

199 / 200
У чоловіка 28-ми років на фоні бронхопневмонії розвинулася кома. Об’єктивно: ЧСС122 /хв, екстрасистолія. АТ- 80/45 мм рт.ст. Шкіра суха, тургор знижений. Дихання глибоке, шумне, рідке. Різкий запах ацетону. Печінка +5 см. Глікемія - 32 ммоль/л.; рН крові - 7,0. Який розчин буде найбільш ефективним для нормалізації дихання? A 28-year-old man developed a coma against the background of bronchopneumonia. Objectively: heart rate 122/min, extrasystole. Blood pressure - 80/45 mm Hg. Skin dry, low turgor. Breathing is deep, noisy, thin. Sharp smell of acetone. Liver +5 cm. Glycemia - 32 mmol/l.; blood pH - 7.0. Which solution will be most effective for normalizing breathing?

0,9% розчин натрію хлориду 0.9% sodium chloride solution

Реополіглюкін Rheopoliglyukin

1% розчин калію хлориду 1% potassium chloride solution

4,2% розчин натрію бікарбонату 4.2% sodium bicarbonate solution

5% розчин глюкози 5% glucose solution

200 / 200
У хлопчика 10-ти років різана рана передньої поверхні передпліччя. З рани витікає тонка цівка темної крові. Який метод тимчасової зупинки кровотечі слід застосувати? A 10-year-old boy has a cut wound on the front surface of his forearm. A thin stream of dark blood flows from the wound. What method should be used to temporarily stop the bleeding?

Пальцеве притиснення плечової артерії Finger compression of the brachial artery

Накласти джгут нижче рани Put a tourniquet below the wound

Накласти джгут вище рани Apply a tourniquet above the wound

Накласти тиснучу пов’язку Apply pressure bandage

Використати метод перерозгинання кінцівки Use the method of overextending the limb